Download as pdf or txt
Download as pdf or txt
You are on page 1of 53

English Medium

VISION IAS
Test -21 TEST BOOKLET
GENERAL STUDIES (P) 20 21 – Test – 3198
Test Booklet Series

C
Time Allowed: Two Hours Maximum Marks: 200

India Economy (I)


National Income Accounting Money and Banking
Definition Money: Definition, Functions and
Concepts: GDP, GNP etc. significance, Evolution of Money, Money
Measurement of national income supply in India and factors affecting it
Money and Banking Banking:
Function of Money, Evolution of Money Need of banks, Function, Commercial
etc. Banks
Banking: Need of Banks, Functions, Central Bank: Importance, functions,
Types, Banking Reforms, Central Bank Structure, Policy tools, Monetary
and its role Repo, MSF etc.)
Policy (CRR, SLR, OMO, Repo and Rev
Inflation and Employment
Definition, types etc. Inflation and Employment
Government Budgeting and Fiscal Policy Government budgeting and Fiscal Policy
Budgeting, types, its evolution in Indi
Process, Component etc.
Public finance, Revenue and Expenditure, + Current Affairs
Capital Market
Terms and Concepts (December 2020)
1. Consider the following statements about the 4. The GDP of a country X is less than its
Regional Rural Banks: GNP. Which is the most appropriate
1. The largest share of its equity is held by deduction from the given statement?
the state government. (a) Income earned within X by foreign
2. One of its objectives is to check the corporations exceeds income by
outflow of rural deposits to urban areas. domestic corporations outside of X.
3. Its amount in agricultural credit (b) Income earned by domestic corporations
disbursement has been steadily outside of X exceeds income earned
increasing since the last five years. within X by foreign corporations.
Which of the statements given above is/are (c) There are more foreign corporations than
correct? domestic corporations in X.
(a) 1 only (d) There are more domestic corporations
(b) 1 and 2 only than foreign corporations in X.
(c) 2 and 3 only
(d) 1, 2 and 3 5. In the context of the participatory notes in
the Indian economy, consider the following
2. With reference to Incremental Capital statements:
Output Ratio (ICOR), consider the following 1. It is a derivative instrument issued in
statements. foreign jurisdictions.
1. It is the ratio of annual investment to the 2. This instrument was introduced to curb
annual increase in GDP. the menace of black money.
2. High ICOR implies a more efficient 3. They are regulated by SEBI.
economy. Which of the statements given above is/are
Which of the statements given above is/are correct?
correct? (a) 1 only
(a) 1 only (b) 1 and 3 only
(b) 2 only (c) 2 only
(c) Both 1 and 2 (d) 2 and 3 only
(d) Neither 1 nor 2
6. Recently the National Common Mobility
3. Which of the following mechanism can be Card (NCMC) was in the news. In this
done to reduce the menace of 'Non- context, an individual in India carrying an
Performing Asset' (NPA) in India? NCMC can use it to pay for
1. Selling of NPAs 1. Metro or bus travel
2. Approaching Lok Adalats 2. Toll on highways
3. Infusing more capital into Banks 3. Retail purchase
Select the correct answer using the code Select the correct answer using the code
given below. given below.
(a) 1 only (a) 1 and 2 only
(b) 2 and 3 only (b) 2 and 3 only
(c) 1 and 2 only (c) 1 only
(d) 1, 2 and 3 (d) 1, 2 and 3

2 www.visionias.in ©Vision IAS


7. Consider the following statements with 10. Which of the following does not attract a
reference to Cost Push inflation: Goods and Service Tax?
1. Cost-push inflation is determined by 1. Solar Power
supply-side factors such as higher wages 2. Milk
etc. 3. Print Media Advertisements
2. Cost-push inflation can only occur when 4. School Education
demand is relatively inelastic. Select the correct answer using the code
Which of the statements given above is/are given below.
correct? (a) 1, 2 and 4 only
(a) 1 only (b) 3 and 4 only
(b) 2 only (c) 1 and 2 only
(c) Both 1 and 2 (d) 1, 2, 3 and 4
(d) Neither 1 nor 2
11. Purchasing Power Parity and market
8. Consider the following statements. exchange rates are the two most common
1. GDP Deflator takes into account all the methods for comparing GDP of countries.
goods and services which are produced With reference to this, consider the
in a country, while CPI does not. following statements.
2. CPI includes the prices of imported 1. PPP rates are relatively stable over time
goods, while GDP Deflator does not. while market rates are more volatile.
3. The weights assigned to goods and 2. India has a greater share in the world
services are constant in GDP Deflator, economy using PPP rates than market-
while it is not in CPI. based rates.
Which of the statements given above are Which of the statements given above is/are
correct? correct?
(a) 1, 2 and 3 (a) 1 only
(b) 1 and 2 only (b) 2 only
(c) 1 and 3 only (c) Both 1 and 2
(d) 2 and 3 only (d) Neither 1 nor 2

9. Which of the following parameters are 12. Fiscal Responsibility and Budget
included in the Human Development Index Management (FRBM) Act provides for
(HDI), published by the United Nations escape clause on which of the following
Development Program (UNDP)? grounds?
1. GNI per capita 1. National security
2. Expected years of schooling 2. Collapse of agricultural output
3. Under five mortality rate 3. Dispute with trading partners
4. Mean years of schooling 4. National calamity
Select the correct answer using the code Select the correct answer using the code
given below. given below.
(a) 1, 2 and 3 only (a) 2 and 3 only
(b) 3 and 4 only (b) 1, 2 and 4 only
(c) 1, 2 and 4 only (c) 1 and 4 only
(d) 1, 2, 3 and 4 (d) 1, 2, 3 and 4
3 www.visionias.in ©Vision IAS
13. Consider the following statements about the 16. Which of the following steps has/have been
‘International Association of Insurance taken by the government for the
Supervisors’ (IAIS): formalization of the labour market?
1. It is the international standard-setting 1. Introduction of Universal Account
Number.
body for the insurance sector.
2. Contribution of 12 % by the government
2. It was established by the World Bank.
to the Employees’ Provident Fund
3. No Indian entity is a member of IAIS. 3. Introduction of the National Career
Which of the statements given above is/are Service (NCS) Project
correct? Select the correct answer using the code
(a) 1 only below.
(b) 1 and 2 only (a) 3 only
(c) 3 only (b) 1, 2 and 3
(c) 2 and 3 only
(d) 1, 2 and 3
(d) 1 and 2 only

14. In the context of Indian economy, which of


17. Consider the following statements.
the following statements is/are correct? 1. A base year is used for comparison in
1. GDP growth rate of India has always the measure of economic indices like
been directly proportional to the rate of GDP or CPI.
growth of employment since the 1991 2. A change in the base year is essential to
reforms. track structural changes in an economy.
2. The rate of growth of employment has 3. Currently, in India the base year for
GDP calculation is 2011-12 and for CPI
always been lower than the rate of GDP
calculation is 2010.
growth.
Which of the statements given above is/are
Select the correct answer using the code correct?
given below. (a) 1, 2 and 3
(a) 1 only (b) 1 and 3 only
(b) 2 only (c) 1 and 2 only
(c) Both 1 and 2 (d) 1 only
(d) Neither 1 nor 2
18. Pooja who is a Ph.D. scholar was working as
15. Consider the following statements based on an clerk in the government school. But after
working for six months, she realized that this
released data of Periodic Labor Force Survey
job is not suitable for her and she resigned
2017-18 with respect to the available data of
from the job. Then, she filled form for the
2011-12: post of assistant professor whose exam is
1. The proportion of women workers in the gonging to be conducted next month. In the
‘salaried’ employees’ category has above passage, Pooja faced which types of
increased during this period. unemployment?
2. There is a decline in casual labor who 1. Seasonal unemployment
mainly originated from the rural sectors 2. Frictional unemployment
3. Underemployment
during this period.
4. Structural unemployment
Which of the statements given above is/are
Select the correct answer using the code
correct? given below.
(a) 1 only (a) 1, 2 and 3 only
(b) 2 only (b) 1, 3, and 4 only
(c) Both 1 and 2 (c) 2 and 3 only
(d) Neither 1 nor 2 (d) 2 and 4 only
4 www.visionias.in ©Vision IAS
19. Which of the following statement is correct 23. Which of the following is/are the
regarding the 'Capital Adequacy Ratio'? instruments available with IMF to
(a) It is a parameter to tackle the effect of
monetarily assist the member countries?
inflation.
(b) It came up through an agreement which 1. Reserve Tranche Position
is known as Basel accord. 2. Contingent Reserve Arrangement
(c) It is same for banks across India.
3. Project Preparation Special Fund
(d) The Primary dealers and NBFCs are
exempted from this. Select the correct answer using the code
given below.
20. Consider the following statements: (a) 1 and 2 only
1. Wage rates for workers under the
(b) 2 and 3 only
MGNREGA are linked with Consumer
Price Index- Rural Labourers (CPI-RL). (c) 1 only
2. Consumer Price Index Numbers for (d) 1, 2 and 3
Rural Labourers (CPI-RL) is published
by the Ministry of Rural Development.
24. Which of the following activities can be
Which of the statements given above is/are
correct? performed by the Small Financial Banks?
(a) 1 only 1. Supplying credits to small and marginal
(b) 2 only
farmers
(c) Both 1 and 2
(d) Neither 1 nor 2 2. Supplying credits to micro and small
industries
21. Arrange the following sectors of Index of 3. Being a business correspondent (BC) for
Industrial Production (IIP) under the used another bank
based classification in decreasing order.
1. Primary goods Select the correct answer using the code
2. Consumer durable goods given below.
3. Capital goods (a) 1 only
Select the correct answer using the code
(b) 1 and 2 only
given below.
(a) 1-3-2 (c) 2 and 3 only
(b) 3-1-2 (d) 1, 2 and 3
(c) 2-1-3
(d) 1-2-3
25. Consider the following statements:

22. With reference to fiat money in India, 1. Product Taxes and Subsidies are
consider the following statements: independent of volume of production.
1. It is backed by the guarantee of the 2. Production Taxes and Subsidies are paid
Central Government.
or received per unit or product.
2. It is a virtual or digital currency that
makes use of encryption techniques. Which of the statements given above is/are
Which of the statements given above is/are correct?
correct? (a) 1 only
(a) 1 only
(b) 2 only
(b) 2 only
(c) Both 1 and 2 (c) Both 1 and 2
(d) Neither 1 nor 2 (d) Neither 1 nor 2

5 www.visionias.in ©Vision IAS


26. In the context of economics, what are 29. House Price Index (HPI) is published by
Negative interest rates? (a) National Housing Bank
(a) It is an unconventional policy used by (b) Ministry of Housing & Urban Affairs
Central Bank to push up inflation rates (c) NITI Aayog
(d) Reserve Bank of India
by reducing interest rates past zero.
(b) Negative interest rates is an
30. With reference to the ‘Apex Committee for
unconventional policy used by Central
the Implementation of the Paris Agreement’
Banks to attract foreign investors.
(AIPA)' consider the following statements:
(c) Negative interest rates is an 1. It has been constituted under
unconventional policy used by Central Environment (Protection) Act, 1986.
Banks to control high inflation rates. 2. Minister of Environment, Forest, and
(d) Negative interest rates is an Climate Change is the chairperson of the
unconventional policy used by Central committee.
Banks to increase the cost of borrowing. 3. It functions as a national authority to
regulate carbon markets in India.
Which of the statements given above are
27. Consider the following statements regarding
correct?
Gross Capital Formation (GCF):
(a) 1 and 2 only
1. It is the outlay on addition to fixed assets (b) 1 and 3 only
including the net change in inventories. (c) 2 and 3 only
2. Increase in GCF may lead to rise in (d) 1, 2 and 3
GDP.
3. GCF of India was lower than GCF of 31. The situation experienced in an economy
Bangladesh in 2019. when there is a sudden and temporary
Which of the statements given above is/are increase in the prices of goods due to the
correct? steps being taken by the government to
revive the economy from the recession is
(a) 2 only
called
(b) 1 and 2 only
(a) Stagflation
(c) 1 and 3 only
(b) Reflation
(d) 1, 2 and 3 (c) Skewflation
(d) Disinflation
28. Which of the statements best describes
“skewflation”? 32. Arrange the following taxes in increasing
(a) It refers that there is meagre rise of order of their respective collection (actuals)
inflation in the economy due to slow as per Budget 2020-21.
economic growth. 1. Union Excise Duties
(b) It is measure of inflation by excluding 2. Corporation Tax
3. Central Goods and Services Tax (CGST)
commodities whose prices are more
4. Securities Transaction Tax
volatile.
Select the correct answer using the code
(c) It is a situation in which price levels of
given below.
few goods increases at a faster rate (a) 4-2-3-1
compared to others. (b) 4-1-2-3
(d) It is inflation caused by the changes in (c) 1-4-3-2
monetary policy. (d) 4-1-3-2

6 www.visionias.in ©Vision IAS


33. The Medium-Term Fiscal Policy Statement 36. Consider the following:
cum Fiscal Policy Strategy Statement is 1. Loans
presented to Parliament under Section 3 of
2. Reserves
the Fiscal Responsibility and Budget
3. Deposits
Management Act, 2003. It sets out the three-
4. Holdings of Debt Securities
year rolling targets for which of the
following? Which of the given above is/are the assets of
1. Fiscal Deficit a commercial bank?
2. Foreign Exchange Reserves (a) 1 only
3. Current Account Deficit
(b) 1, 2 and 4 only
4. Central Government Debt
(c) 3 and 4 only
Select the correct answer using the code
(d) 1, 2 and 3 only
given below.
(a) 1, 2 and 3 only
(b) 2 and 3 only 37. Vazhakulam town was recently seen in the
(c) 1 and 4 only news. It is most famous for:
(d) 1, 2, 3 and 4 (a) Wooden toys
(b) Cotton sarees
34. Which of the following correctly describes
(c) Pineapples
the phenomenon of Fiscal Drag?
(d) Rice
(a) Widening of government deficit due to
dependency on borrowings for fueling
economic growth. 38. In India, retail inflation based on the
(b) Movement of taxpayers into higher tax consumer price index (CPI) jumped to an
brackets owing to inflation. eight-month high of 7.34 percent in
(c) Lower purchasing power of domestic
September 2020 which is primarily driven
currency due to higher dependency on
by higher food inflation. Which of the
imports.
(d) Loss of government revenue due to the following steps can be helpful to counter
emigration of people. food inflation?
1. Imposing the Minimum Export Price
35. Consider the following statements with (MEP) on selected food items.
reference to monetisation of deficit: 2. Maintenance of buffer stock by the
1. It involves financing of such extra
government.
expenses by borrowing money from the
3. Providing the credit link subsidies for
market.
2. It is a type of debt financing. establishing Mega Food parks.
3. In India, it is currently not allowed. Select the correct answer using the code
Which of the statements given above is/are given below.
correct? (a) 1 and 2 only
(a) 1 and 2 only
(b) 2 and 3 only
(b) 2 and 3 only
(c) 1 and 3 only
(c) 3 only
(d) 1 and 3 only (d) 1, 2 and 3

7 www.visionias.in ©Vision IAS


39. With reference to 'Aadhar enabled payment 42. Which of the following contribute to the
systems' (AEPS), consider the following Gross Domestic Product (GDP) of India?
statements: 1. Sale of car A, owned by Indian
1. It will speed up the process of financial Company and manufactured in India.
inclusion in the country. 2. Sale of car B, owned by Indian
2. It will reduce the use of ATMs. Company and manufactured in Britain.
3. With AEPS, the only information 3. Sale of car C, owned by Korean
required to initiate a transaction will be
Company and manufactured in India.
the Aadhaar number and biometric
Select the correct answer using the code
information.
given below.
Which of the statements given above is/are
(a) 1, 2, and 3
correct?
(b) 1 and 2 only
(a) 1 only
(c) 2 and 3 only
(b) 1 and 2 only
(d) 1 and 3 only
(c) 2 and 3 only
(d) 1, 2 and 3
43. The term 'Jurisdictions Under Increased
Monitoring' often seen in news is related to
40. Which of the following countries are the
founding members of the New Development which of the following organistion?

Bank? (a) Bank of International Settlement

1. Brazil (b) World Bank


2. Philippines (c) The Financial Action Task Force
3. India (d) International Monetary Fund
4. Canada
5. South Africa 44. Consider the following differences between
Select the correct answer using the code core and headline inflation:
given below. 1. Core Inflation reflects the rate of change
(a) 1, 2 and 3 only in prices of all goods and services in an
(b) 3, 4 and 5 only economy, whereas headline inflation
(c) 2, 4 and 5 only excludes items that face volatile price
(d) 1, 3 and 5 only movement.
2. Generally, core inflation is the preferred
41. In which of the following groups are all the
tool for Central Banks to frame long-
four countries members of the Shanghai
term policy.
Cooperation Organisation?
Which of the statements given above is/are
(a) Russia, Kazakhstan, Turkmenistan and
correct?
India
(a) 1 only
(b) India, China, Tajikistan and Kyrgyzstan
(b) 2 only
(c) Mongolia, Russia, Pakistan and China
(c) Both 1 and 2
(d) Pakistan, Uzbekistan, India and
(d) Neither 1 nor 2
Afghanistan
8 www.visionias.in ©Vision IAS
45. Which of the following statements is correct 48. Immediately after a nation's currency is
regarding the term 'liquidity trap'? devalued, imports get more expensive and
(a) It is a situation when money gets held in exports get cheaper, worsening trade deficit.
the banking system due to technical Shortly thereafter, the sales volume of the
nation's exports begins to rise steadily and at
issues.
the same time, consumers at home begin to
(b) In this situation the interest rate gets
buy more locally-produced goods because
increases manifold. they are relatively affordable compared to
(c) Open Market Operation helps in imports leading to better trade balance if not
overcoming this situation. trade surplus. Which of the following best
(d) None of the above suit has the description given above?
(a) J-curve effect
46. Which of the following step(s) can be taken (b) Laffer effect
(c) W-curve effect
by the government to reduce the inflationary
(d) Kuznets effect
gap?
1. Increasing the salaries of government
49. With reference to the term ‘Emergency use
employees.
authorization’ (EUA) recently seen in the
2. Enhancing the minimum support price news, consider the following statements:
(MSP) of the notified agricultural 1. It allows medicine or vaccine to be used
commodities. on the public till the completion of final
3. Issuing of new dated securities by the trials.
government. 2. India’s drug regulations do not have
Select the correct answer using the code provisions for a mechanism like
'Emergency use authorization'.
given below.
Which of the statements given above is/are
(a) 1 and 3 only
correct?
(b) 1 and 2 only (a) 1 only
(c) 2 and 3 only (b) 2 only
(d) 3 only (c) Both 1 and 2
(d) Neither 1 nor 2
47. India plans dam on Brahmaputra to offset
Chinese construction upstream. In this 50. The Statutory Liquidity Ratio which a bank
context, which of the following statements has to maintain is composed of
is/are correct about the ‘Brahmaputra River’ 1. Instruments as notified by the RBI
2. Gold
in India?
3. Treasury-Bills of the Government of
1. Its basin possesses about 75% of India’s
India
total hydropower potential. 4. State Development Loans (SDLs) issued
2. It accounts for more than 50% of the by State Governments
total run-off of India’s rivers. Select the correct answer using the code
Select the correct answer using the code given below.
given below. (a) 1 and 3 only
(a) 1 only (b) 2 and 4 only
(c) 1, 2 and 3 only
(b) 2 only
(d) 1, 2, 3 and 4
(c) Both 1 and 2
(d) Neither 1 nor 2
9 www.visionias.in ©Vision IAS
51. Which of the following organisations 55. Private Final Consumption Expenditure
released the “World Employment and Social (PFCE) is used in the calculation of GDP of
Outlook: Trends 2020”? India through the expenditure approach.
(a) International Labour Organisation PFCE includes the final consumption
(b) World Economic Forum expenditure of:
(c) International Monetary Fund (a) Households only
(d) The World Bank (b) Households and Non Profit Institutions
Serving Households
52. Which of the following will not lead to an (c) Households, Non Profit Institution
increase in forex reserves of a country? Serving Households and Business
(a) Increase in exports Entities
(b) Increase in total world trade (d) Non Profit Institution Serving
(c) Increase in inbound international Households and Business Entities
tourism.
(d) Increase in Foreign Direct Investments 56. In the recent COVID crisis, various steps
have been taken to revive the economy.
53. Consider the following statements: Which of the following steps could lead to
1. Value of Personal Disposable Income is an increase in the burden on the state
always more than Personal Income. exchequer in form of the fiscal deficit?
2. Personal Disposable Income is the part 1. Loan Moratorium
of the aggregate income which belongs 2. Monetary support to poor people
to the households. 3. Investment in vaccine research
Which of the statements given above is/are Select the correct answer using the code
correct? given below.
(a) 1 only (a) 1 and 2 only
(b) 2 only (b) 2 and 3 only
(c) Both 1 and 2 (c) 1 and 3 only
(d) Neither 1 nor 2 (d) 1, 2 and 3

54. With reference to the ‘Nuclear Magnetic 57. Consider the following statements regarding
Resonance (NMR) spectroscopy’ consider GVA (Gross Value Added) and GDP (Gross
the following statements: Domestic Product):
1. It is the study of molecules by recording 1. While GVA depicts the state of
the interaction of radiofrequency (Rf) economic activity from the producers'
electromagnetic radiations with the side, GDP depicts the demand
nuclei of molecules placed in a strong perspective.
magnetic field 2. GVA is a better reflection of
2. It can be used to determine the purity of productivity than GDP as it excludes
samples of milk. indirect taxes.
Which of the statements given above is/are Which of the statements given above is/are
correct? correct?
(a) 1 only (a) 1 only
(b) 2 only (b) 2 only
(c) Both 1 and 2 (c) Both 1 and 2
(d) Neither 1 nor 2 (d) Neither 1 nor 2

10 www.visionias.in ©Vision IAS


58. Which of the following statements are 61. With reference to the Cash Reserve Ratio in
correct regarding the functions of the India (CRR), consider the following
statements:
Reserve Bank of India?
1. This varies as per an individual bank’s
1. It facilitates external trade and payment. financial situation and size.
2. It introduces and upgrades safe and 2. All Scheduled Commercial Banks are
efficient modes of payment systems in required to maintain the CRR.
3. Banks are paid a timely interest by the
the country.
RBI if they are successful in maintaining
3. It performs merchant banking function the CRR.
for state governments. Which of the statements given above is/are
4. It maintains banking accounts of all correct?
(a) 1 only
scheduled banks.
(b) 2 and 3 only
Select the correct answer using the code (c) 2 only
given below. (d) 1, 2 and 3
(a) 1 and 2 only
(b) 2, 3 and 4 only 62. Which of the following can be considered as
transfer payments?
(c) 1, 2, 3 and 4
1. Payment of MGNREGA wages to a
(d) 3 and 4 only beneficiary.
2. Payment of subsidy to a farmer under
59. With reference to the Consumer Price Index, PM Kisan Yojana.
3. Payment of pension to a beneficiary
which of the following item has been given
under the National Old Age Pension
the highest weightage? scheme.
(a) Fuel and Light Select the correct answer using the code
(b) Housing given below.
(a) 1, 2 and 3
(c) Clothing and Footwear
(b) 1 only
(d) Food and Beverages (c) 1 and 3 only
(d) 2 and 3 only
60. Which of the following measures should be
taken when an economy is witnessing 63. Which of the following statements best
describes the term 'Labour Force
inflationary pressures?
Participation Rate (LFPR)' in India?
1. The interest rate can be decreased. (a) It is the percentage of persons
2. The direct taxes can be increased. unemployed among the persons in the
3. The public spending can be reduced. labour force.
(b) It is the percentage of employed persons
Select the correct answer using the code
in the population.
given below.
(c) It is the percentage of labour force
(a) 2 and 3 only (working or seeking for work) in the
(b) 2 only population divided by the total working-
age population.
(c) 1 and 3 only
(d) None of the above.
(d) 1, 2, and 3
11 www.visionias.in ©Vision IAS
64. It is the economic situation, in which, there 67. Output-Outcome Monitoring Framework
is a drop in the price of the commodities (OOMF), sometimes seen in the news, is
over a period of time, along with a higher related to
level of unemployment and investor prefers
(a) WTO Negotiations over Agreement on
to buy bonds over the equity. In order to deal
Agriculture (AoA)
with this situation, the government adopts
the ‘expansion monetary policy’. (b) Clinical trials of COVID Vaccine
Which form of economic situation is being (c) Greenhouse Gas Emissions
described in the above passage? (d) Central Sector (CS) and Centrally
(a) Disinflation Sponsored Schemes
(b) Deflation
(c) Stagflation
68. Periodic Labour Force Survey (PLFS),
(d) Structural Inflation
released by

65. Which among the following statements (a) Ministry of Labour and Employment
regarding Market Stabilisation Scheme (b) Ministry of Agriculture & Farmers
(MSS) is not correct? Welfare
(a) The securities issued under MSS are (c) NITI Aayog
issued by way of auctions conducted by
(d) Ministry of Statistics and Programme
the Government of India.
Implementation
(b) Tools like the Liquidity Adjustment
Facility (LAF) and Open Market
Operations (OMO) perform the same 69. Which of the following represents National
task as the MSS. Income?
(c) The amount raised under the MSS are (a) Net National Product at factor cost
used only for the purpose of buying back (b) Net National Product at market prices
of Treasury-Bills issued under the
(c) Gross National Product at factor cost
scheme.
(d) Gross National Product at market prices
(d) It is use to absorb excess liquidity from
the market
70. Arrange the following heads in increasing
66. Consider the following sectors. order of revenue expenditures as per
1. Construction Budget 2020-21.
2. Manufacturing 1. Interest Payment and Servicing of Debt
3. Finance, Real estate & Professional
2. Defence Services
Services
3. Agriculture and Allied Activities
4. Agriculture, Forestry and Fishing
4. Transport
Which of the following is the correct
sequence of decreasing order of sectoral Select the correct answer using the code
shares in GVA in 2018-19? given below.
(a) 3-2-1-4 (a) 2-1- 3 - 4
(b) 2-1-3-4 (b) 2 - 1 - 4 - 3
(c) 3-2-4-1
(c) 1 - 4 - 2 - 3
(d) 2-1-4-3
(d) 2 - 4 - 1 - 3
12 www.visionias.in ©Vision IAS
71. Which of the following regarding Gross 75. Consider the following statements in the
Value Added (GVA) is correct? context of Ninth Edition of the Status Paper
(a) GVA at market prices equals GVA at on the Government Debt:
1. Most of the Government debt is at fixed
factor cost plus net production taxes.
interest rates.
(b) GVA at factor cost equals GVA at basic 2. The share of short-term debt has
prices plus net product taxes. stabilised after some rising.
(c) GVA at basic prices equals GVA at Which of the statements given above is/are
factor costs plus net production taxes correct?
(d) GVA at market prices equals GVA at (a) 1 only
(b) 2 only
basic prices plus net production taxes.
(c) Both 1 and 2
(d) Neither 1 nor 2
72. Consider the following statements:
1. Higher interest rate leads to lesser 76. The term ‘Special and Differential
demand of money. Treatment (S&DT)’ is often seen in the news
2. Interest rate is decided by the Central in reference to:
Government. (a) Forest rights of Scheduled tribes and
other forest dwellers
Which of the statements given above is/are
(b) Negotiation under Paris Climate
correct? Agreement
(a) 1 only (c) Special provisions of WTO for the
(b) 2 only developing countries
(c) Both 1 and 2 (d) Special provisions for protecting the
(d) Neither 1 nor 2 interests of economically weaker
sections

73. Which of the following defines 'Cash


77. 'Neom city’ often seen in the news, is a
Deposit Ratio'? futuristic megacity that will be constructed
(a) Ratio of money held by the public in in which of the following countries?
currency to that they hold in bank (a) Norway
deposits. (b) India
(b) Proportion of the total deposits (c) Saudi Arabia
(d) New Zealand
commercial banks keep as reserves.
(c) Ratio of cash held by the banks in
78. Which of the following can be the reason/
comparison to gold.
reasons for cost-push inflation?
(d) Ratio of Cash Reserve Ratio to Statutory 1. Enhancement in the price of the raw
Liquidity Ratio fixed by RBI. materials.
2. Adoption of expansionary monetary
74. The 'Joint Comprehensive Plan of Action' policy by the Reserve Bank of India
(RBI).
often seen in the news is related to
3. Increase in wages of employees.
(a) a global agreement on climate change Select the correct answer using the codes
(b) India-Iran-Afghanistan trade and transit given below.
agreement (a) 2 only
(c) Iran Nuclear Deal (b) 1 and 2 only
(d) a UN led initiative to curb money (c) 1 and 3 only
(d) 1, 2 and 3
laundering activities

13 www.visionias.in ©Vision IAS


79. In the context of banking in India, the term 82. Which of the following is/are the
'spread' describe as Development Financial Institutions in India?
(a) It is the difference in the interest rates 1. Reserve Bank of India
between that paid by the banks to 2. National Bank for Agriculture and Rural
depositors and the rate charged from the Development
borrowers. 3. National Housing Bank
(b) It is the difference in the rate of interest Select the correct answer using the code
between Repo and Reverse repo rate. given below.
(c) It is the ratio between assets and (a) 1 only
(b) 1 and 3 only
liabilities of commercial bank.
(c) 1 and 2 only
(d) None of the above
(d) 2 and 3 only

80. With reference to the Harmonized System of


83. With reference to Non-Banking Financial
Nomenclature (HSN) under the Goods and
Companies (NBFCs), consider the following
Service Tax, which of the following
statements:
statements is not correct?
1. NBFCs can be registered either with
(a) It is a unique code to identify the SEBI or RBI.
different goods and services. 2. It can issue cheques drawn on itself.
(b) It was developed by the World Customs Which of the statements given above is/are
Organization (WCO). correct?
(c) HSN Code is mandatory to be (a) 1 only
mentioned in GST invoices. (b) 2 only
(d) It facilitates the uniform classification of (c) Both 1 and 2
products and smooth international trade. (d) Neither 1 nor 2

81. Consider the following pairs: 84. A part of the labour force is either left
Term Meaning without work or is working in a redundant
1. Hard : It is the international manner such that worker productivity is
currency currency in which the essentially zero. This situation best describes
highest faith is shown which of the following types of
and is needed by unemployment?
every economy. (a) Hidden unemployment
: It is used to denote the (b) Structural unemployment
domestic currency (c) Cyclical unemployment
2. Heated (d) Voluntary unemployment
which is under enough
currency
pressure of
depreciation. 85. Which of the tools given below are used by
3. Dear money : It denotes a period of RBI to control liquidity and inflation?
comparatively 1. Open Market Operations
higher/costlier interest 2. Bank Rate
3. Repo rate and Reverse repo rate
rates regime.
4. Increasing the tax base
Which of the pairs given above is/are
Select the correct answer using the code
correctly matched?
given below.
(a) 1 only
(a) 1 and 2 only
(b) 1 and 2 only
(b) 1, 2 and 3 only
(c) 2 and 3 only
(c) 3 and 4 only
(d) 1, 2 and 3 (d) 1, 2, 3 and 4
14 www.visionias.in ©Vision IAS
86. A higher level of GDP cannot be regarded as 89. In the context of economics, the Phillips
an index of greater wellbeing of the people curve states that
of a country. Which of the following are the (a) inflation results into creation of
reasons for this? employment.
1. Distribution of GDP may not be (b) with the rise of tax rate, the amount of
uniform. tax revenue collected by government
2. Growth of GDP results in the rise of increases upto a certain limit.
Direct taxes. (c) inflation results into stagnant economic
3. Many activities in an economy are not growth.
evaluated in monetary terms. (d) there is a positive relationship between
4. GDP inherently does not take into rates of employment and GDP growth.
account negative externalities.
Select the correct answer using the code 90. With respect to Electronically Transmitted
given below. Postal Ballot System (ETPBS), consider the
(a) 1, 2, 3 and 4 following statements:
(b) 2 and 3 only 1. ETPBS is developed by Bharat
(c) 1, 3 and 4 only Electronics Limited and Electronics
(d) 1 and 4 only Corporation of India Limited.
2. It enables the entitled voters to cast their
87. Which of the following lakes are found in vote using an electronically received
Ladakh region? postal ballot from anywhere outside their
1. Pangong Tso lake constituency.
2. Chandra taal lake Which of the statements given above is/are
3. Tso kar lake correct?
4. Tsomoriri lake (a) 1 only
Select the correct answer using the code (b) 2 only
given below. (c) Both 1 and 2
(a) 1 and 3 only (d) Neither 1 nor 2
(b) 2 and 4 only
(c) 1, 3 and 4 only 91. Consider the following statements with
(d) 1, 2, 3 and 4 reference to the measurement of Inflation:
1. Whereas CPI and WPI are based on a
88. Consider the following statements with limited basket of goods and services,
reference to the Municipal bonds: GDP deflator reflects the prices of all
1. Any urban local body of the country can domestically produced goods and
issue such bonds. services in the economy.
2. These bonds could be issued for 2. Monthly change in inflation can be best
infrastructure projects only. tracked using the GDP deflator.
Which of the statements given above Which of the statements given above is/are
is/are not correct? correct?
(a) 1 only (a) 1 only
(b) 2 only (b) 2 only
(c) Both 1 and 2 (c) Both 1 and 2
(d) Neither 1 nor 2 (d) Neither 1 nor 2

15 www.visionias.in ©Vision IAS


92. If the economy is booming, which of the 96. Which of the following initiative(s) will help
following steps would amount to a to improve female work participation?
countercyclical approach to fiscal policy? 1. Setting-up of One Stop Centre in all
1. Reducing government spending districts of the country.
2. Raising tax rates 2. Prime Minister’s Employment
3. Increasing interest rates Generation Programme.
Select the correct answer using the code 3. Deendayal Antyodaya Yojana National
given below. Rural Livelihoods Mission.
(a) 1 and 2 only 4. Rashtriya Mahila Kosh.
(b) 2 and 3 only Select the correct answer using the code
(c) 1 and 3 only below.
(d) 1, 2 and 3 (a) 1, 2 , 3 and 4
(b) 2 and 3 only
93. Usual status unemployment is calculated (c) 1 and 2 only
with reference to a period of (d) 1, 3 and 4 only
(a) one year
(b) six months 97. Consider the following statements about
(c) one week Foreign Portfolio Investors (FPI):
(d) one day 1. Investment here by any single investor
or investor group cannot exceed 10% of
the equity of an Indian company.
94. Which of the following can be advantage(s)
2. They can invest in both the listed and
of inflation in developing economies?
unlisted shares.
1. It favours Foreign Portfolio Investments
3. Foreign Portfolio Investment is a part of
(FPI) in the market.
a country's capital account.
2. Inflation can be favorable for economic
Which of the statements given above is/are
growth.
correct?
Select the correct answer using the code
(a) 1 only
given below.
(b) 1 and 3 only
(a) 1 only
(c) 2 only
(b) 2 only
(d) 2 and 3 only
(c) Both 1 and 2
(d) Neither 1 nor 2
98. "It is a thousand-year-old traditional paper
making technique called 'Mon shugu' or the
95. With reference to the recent merging of
paper of the Monpa people. It is made using
banks in India, consider the following
the bark of the shugu sheng shrub (Daphne
statements:
papyracea). For centuries, the paper has been
1. The decision to merge the Public Sector
used for religious scriptures, manuscripts,
Banks falls under the ambit of the RBI.
prayer flags, and sometimes as part of flag
2. In a merger, two or more banks are
poles and prayer wheels."
merged to form a new entity.
To which of the following states/UT does
Which of the statements given above is/are
the above described paper making technique
correct? belong to?
(a) 1 only (a) Ladakh
(b) 2 only (b) Himachal Pradesh
(c) Both 1 and 2 (c) Arunachal Pradesh
(d) Neither 1 nor 2 (d) Sikkim
16 www.visionias.in ©Vision IAS
99. Recently an Artificial Intelligence and
Robotics Technologies Park (ARTPARK)
was set up in which of the following places
in India?
(a) Bengaluru, Karnataka
(b) Gurgaon, Haryana
(c) Hyderabad, Telangana
(d) Mumbai, Maharashtra

100. With reference to the recently initiated 'Early


Health Warning System' in India, consider
the following statements:
1. It is based on the relationship between
weather changes and disease incidence
in India.
2. It is being developed by the Ministry of
Earth Sciences.
Which of the statements given above is/are
correct?
(a) 1 only
(b) 2 only
(c) Both 1 and 2
(d) Neither 1 nor 2

Copyright © by Vision IAS


All rights are reserved. No part of this document may be reproduced, stored in a retrieval system or transmitted
in any form or by any means, electronic, mechanical, photocopying, recording or otherwise, without prior
permission of Vision IAS.
17 www.visionias.in ©Vision IAS
VISIONIAS
www.visionias.in
ANSWERS & EXPLANATIONS
GENERAL STUDIES (P) TEST – 3198 (2021)

Q 1.C
• The equity of a regional rural bank is held by the Central Government, concerned State Government and
the Sponsor Bank in the proportion of 50:15:35. Hence statement 1 is not correct.
• The main objectives of RRB’s are to provide credit and other facilities‚ especially to the small and
marginal farmers‚ agricultural labourers artisans and small entrepreneurs in rural areas with the objective
of bridging the credit gap in rural areas, checking the outflow of rural deposits to urban areas and reduce
regional imbalances and increase rural employment generation. Hence statement 2 is correct.
• In the year 2014-2015 the amount of agricultural credit disbursement by RRB was Rs 1,02,483 crores
• In the year 2015-2016 the amount of agricultural credit disbursement by RRB was Rs 1,19,260 crores
• In the year 2016-2017 the amount of agricultural credit disbursement by RRB was Rs 1,23,216 crores
• In the year 2017-2018 the amount of agricultural credit disbursement by RRB was Rs 1,41,216 crores
• In the year 2018-2019 the amount of agricultural credit disbursement by RRB was Rs 1,51,258
crores. Hence statement 3 is correct.

Q 2.A
• The incremental capital output ratio (ICOR) is a frequently used tool that explains the relationship
between the level of investment made in the economy and the consequent increase in the gross domestic
product (GDP). ICOR indicates the additional unit of capital or investment needed to produce an
additional unit of output.
• ICOR is a metric that assesses the marginal amount of investment capital necessary for a country or other
entity to generate the next unit of production.
• ICOR can be calculated as: ICOR=Annual Investment/Annual Increase in GDP Hence, statement 1 is
correct.
• A lower ICOR is preferred as it indicates a country's production is more efficient.
• For example, suppose that Country X has an incremental capital output ratio (ICOR) of 10. This implies
that Rs.10 worth of capital investment is necessary to generate Re.1 of extra production. Furthermore, if
country X's ICOR was 12 last year, this implies that Country X has become more efficient in its use of
capital. Hence, statement 2 is not correct.

Q 3.C
• Selling the NPAs to SCs/RCs (securitisation companies/ reconstruction companies) registered under the
SARFAESI Act. SCs/RCs are expected to do a specialized task of recovering and reconstructing the
NPAs thereby reducing the NPAs in the system.
• Lok Adalats: Lok Adalat mechanism offers expeditious, in-expensive and mutually acceptable ways of
settlement disputes. Government has advised the public sector banks to utilize this mechanism to its
fullest potential for recovery in Non-performing Assets (NPAs) cases. Lok Adalats deal with NPAs that
fall under ‘doubtful’ or ‘loss’ categories and cover small loan amounts till ₹ 5 lakhs. As per the RBI
guidelines (2001), both suit filed and non-suit filed cases can be dealt in Lok Adalats. Though it is not a
legal procedure, it is highly effective in settling disputes for small loans.
• Infusing more capital into banks: even if it is based on performance, is a hugely risky move without full
transparency. Apparently, well-performing banks may suddenly show themselves to be worse than
reported. Any move to re-capitalize the banks should only be made once full transparency is achieved. So
it cannot be a measure to tackle the NPA issue.
• Hence option (c) is the correct answer.
1 www.visionias.in ©Vision IAS
Q 4.B
• Gross national product (GNP) is an estimate of total value of all the final products and services turned out
in a given period by the means of production owned by a country's residents. GNP is commonly
calculated by taking the sum of personal consumption expenditures, private domestic investment,
government expenditure, net exports and any income earned by residents from overseas investments,
minus income earned within the domestic economy by foreign residents. Net exports represent the
difference between what a country exports minus any imports of goods and services.
• GNP is related to another important economic measure called gross domestic product (GDP), which takes
into account all output produced within a country's borders regardless of who owns the means of
production. GNP starts with GDP, adds residents' investment income from overseas investments, and
subtracts foreign residents' investment income earned within a country.
• Applying the concepts of GDP and GNP to the scenario given in the question where in a Country X, GDP
is less than its GNP. The most appropriate deduction would be option (b) i.e. Income earned by domestic
corporations outside of X exceeds income earned within X by foreign corporations. Hence, option (b) is
the correct answer.
• In case of option (a) i.e. Income earned within X by foreign corporations exceeds income by domestic
corporations outside of X, its GDP would be more than its GNP.
• As for options (c ) and (d), the mere presence of more domestic corporations or more foreign corporations
cannot decide whether GNP would be more or GDP because GDP or GNP can be decided the amount of
productions by domestic and foreign corporations.

Q 5.B
• A Participatory Note (PN or P-Note) in the Indian context, in essence, is a derivative instrument issued
in foreign jurisdictions, by a SEBI registered FII, against Indian securities—the Indian security
instrument may be equity, debt, derivatives or may even be an index.
• PNs are also known as Overseas Derivative Instruments, Equity Linked Notes, Capped Return Notes,
and Participating Return Notes, etc. Hence statement 1 is correct.
• It is considered a highly ‘safe and lucrative route’ to invest the ‘unaccounted’, ‘even illegal’ money into
the Indian security market for huge profits (during the booming period).
• Experts even imagined that it may be allowing the ‘black money’ of India (stashed away from India
through ‘hawala’ kind of illegal channels and deposited in the tax havens of the world in ‘Swiss Bank’
kind of financial institutions) to get invested back in the market. Again, ‘terrorist organizations’ might
have been using this route, too. Hence statement 2 is not correct.
• PNs are market instruments that are created and traded overseas. Hence, Indian regulators cannot ban the
issue of PNs. However, they can regulated, as SEBI does—when a PN is traded on an overseas exchange,
the regulator in that jurisdiction would be the authority to regulate that trade. Hence statement 3 is
correct.

Q 6.D
• Recent Context: The National Common Mobility Card (NCMC) service for the Delhi Metro’s Airport
Express Line was launched by Prime Minister Narendra Modi via video conferencing.
• Ministry of Housing & Urban Affairs brought to the fore the National Common Mobility Card
(NCMC) which is an automatic fare collection system.
o On March 2019, with the tagline 'One Nation One Card’ NCMC was launched in India.
o The High-level Committee on Deepening of Digital Payments (CDDP) under the chairmanship of
Shri Nandan Nilekani recommended the use of NCMC at all transit locations.
o Dubbed as 'One Nation One Card', the inter-operable transport card allows the holders to also pay
for their metro/bus travel, toll taxes, parking charges, retail shopping and even withdraw
money.
o By 2022, NCMC will become available on the entire Delhi Metro network.
• Hence option (d) is the correct answer.

Q 7.C
• Cost-push inflation occurs when overall prices increase (inflation) due to increases in the cost of wages
and raw materials. Higher costs of production can decrease the aggregate supply (the amount of total
production) in the economy. Since the demand for goods hasn't changed, the price increases from
production are passed onto consumers creating cost-push inflation.

2 www.visionias.in ©Vision IAS


• Cost-push inflation is when supply costs rise or supply levels fall. Either will drive up prices as long
as demand remains the same. Shortages or cost increases in labor, raw materials, and capital
goods create cost-push inflation. Hence, statement 1 is correct.
• Cost-push inflation can only occur when demand is relatively inelastic. Inelastic demand is when
people still buy the good or service even if the price goes up. When demand is elastic, people won't pay
the higher prices. They simply buy less of the good or service. They'll either switch to a slightly different
product or do without it. Hence, statement 2 is correct.

Q 8.B
• In order to compare the GDP figures (and other macroeconomic variables) of different countries or to
compare the GDP figures of the same country at different points of time, we cannot rely on GDPs
evaluated at current market prices. For comparison, we take the help of real GDP. Real GDP is calculated
in a way such that the goods and services are evaluated at some constant set of prices (or constant prices).
Since these prices remain fixed, if the Real GDP changes we can be sure that it is the volume of
production which is undergoing changes. Nominal GDP, on the other hand, is simply the value of GDP at
the current prevailing prices.
• The ratio of nominal GDP to real GDP gives us an idea of how the prices have moved from the base year
(the year whose prices are being used to calculate the real GDP) to the current year. In the calculation of
real and nominal GDP of the current year, the volume of production is fixed. Therefore, if these measures
differ it is only due to change in the price level between the base year and the current year. The ratio of
nominal to real GDP is a well known index of prices. This is called GDP Deflator. Thus GDP deflator =
Nominal GDP/ Real GDP.
• There is another way to measure change of prices in an economy which is known as the Consumer Price
Index (CPI). This is the index of prices of a given basket of commodities which are bought by the
representative consumer. CPI is generally expressed in percentage terms.
• CPI (and analogously WPI) may differ from GDP deflator because :
• The goods purchased by consumers do not represent all the goods which are produced in a country. GDP
deflator takes into account all such goods and services. Hence statement 1 is correct. CPI includes prices
of goods consumed by the representative consumer, hence it includes prices of imported goods. GDP
deflator does not include prices of imported goods. Hence statement 2 is correct. The weights are
constant in CPI – but they differ according to production level of each good in GDP deflator. Hence
statement 3 is not correct.

Q 9.C
• The HDI was created by the United Nations Development Programme (UNDP) to emphasize that people
and their capabilities should be the ultimate criteria for assessing the development of a country, not
economic growth alone. The HDI can also be used to question national policy choices, asking how two
countries with the same level of GNI per capita can end up with different human development outcomes.
• The Human Development Index (HDI) is a summary measure of average achievement in key dimensions
of human development: a long and healthy life, being knowledgeable and have a decent standard of living.
The HDI is the geometric mean of normalized indices for each of the three dimensions.
• The health dimension is assessed by life expectancy at birth, the education dimension is measured
by mean of years of schooling for adults aged 25 years and more and expected years of schooling for
children of school entering age. The standard of living dimension is measured by gross national income
per capita. The HDI uses the logarithm of income, to reflect the diminishing importance of income with
increasing GNI. The scores for the three HDI dimension indices are then aggregated into a composite
index using geometric mean.
• Hence option (c) is the correct answer.

Q 10.A
• Goods and Services Tax (GST) is an indirect tax (or consumption tax) used in India on the supply of
goods and services. It is a comprehensive, multistage, destination-based tax: comprehensive because it has
subsumed almost all the indirect taxes except a few state taxes. Multi-staged as it is, the GST is imposed
at every step in the production process, but is meant to be refunded to all parties in the various stages of
production other than the final consumer and as a destination-based tax, it is collected from point of
consumption and not point of origin like previous taxes.
• The tax came into effect from 1 July 2017 through the implementation of the One Hundred and First
Amendment of the Constitution of India by the Indian government. The GST replaced existing multiple
taxes levied by the central and state governments.
3 www.visionias.in ©Vision IAS
• Goods and services are divided into five different tax slabs for collection of tax - 0%, 5%, 12%, 18% and
28%.
• However, petroleum products, alcoholic drinks, and electricity (including solar power) are not taxed
under GST and instead are taxed separately by the individual state governments, as per the previous tax
system. Hence option 1 is correct.
• Some industries and products were exempted by the government and remain untaxed under GST, such
as dairy products, products of milling industries, fresh vegetables & fruits, meat products, and other
groceries and necessities. Hence option 2 is correct.
• in case of newspaper, sale of newspaper to customers is exempt from GST, however selling of
advertisement space in print media attract 5 and 18 percent GST, depending on the terms of the
contract between the newspaper, advertisement agency and the client. Hence option 3 is not correct.
• Schooling up to higher secondary and most of the services provided to educational institutions
are exempt from taxation under GST. Mid-day meal scheme as well as security, cleaning and
housekeeping services performed in educational institutions up to higher secondary are also exempt from
GST. Services relating to admission and examination up to higher secondary are exempt under
GST. Hence option 4 is correct.

Q 11.C
• One of the two main methods of conversion of GDP uses market exchange rates—the rate prevailing in
the foreign exchange market (using either the rate at the end of the period or an average over the period).
The other approach uses the purchasing power parity (PPP) exchange rate—the rate at which the currency
of one country would have to be converted into that of another country to buy the same amount of goods
and services in each country.
• To facilitate price comparisons across countries, the International Comparisons Program (ICP) was
established by the United Nations and the University of Pennsylvania in 1968. PPPs generated by the ICP
are based on a global survey of prices. To understand PPP, let’s take a commonly used example, the price
of a hamburger. If a hamburger is selling in London for £2 and in New York for $4, this would imply a
PPP exchange rate of 1 pound to 2 U.S. dollars.
• PPP versus market rates
o Advantages of PPP: The main one is that PPP exchange rates are relatively stable over time. By
contrast, market rates are more volatile, and using them could produce quite large swings in
aggregate measures of growth even when growth rates in individual countries are stable. Hence,
statement 1 is correct.
o Another drawback of market-based rates is that they are relevant only for internationally traded goods.
Nontraded goods and services tend to be cheaper in low-income than in high-income countries. A
haircut in New York is more expensive than in Lima; the price of a taxi ride of the same distance is
higher in Paris than in Tunis; and a ticket to a cricket game costs more in London than in Lahore.
Indeed, because wages tend to be lower in poorer countries, and services are often relatively labor
intensive, the price of a haircut in Lima is likely to be cheaper than in New York even when the cost
of making tradable goods, such as machinery, is the same in both countries. Drawbacks of PPP: The
biggest one is that PPP is harder to measure than market-based rates.
o There is a large gap between market- and PPP-based rates in emerging market and developing
countries, for most of which the ratio of the market and PPP U.S. dollar exchange rate is between 2
and 4. But for advanced economies, the market and PPP rates tend to be much closer. As a result,
developing countries get a much higher weight in aggregations that use PPP exchange rates than they
do using market exchange rates. The weights of China and India in the world economy are far
greater using PPP exchange rates than market-based weights. Hence, statement 2 is correct.

Q 12.B
• Fiscal Responsibility and Budget Management (FRBM) became an Act in 2003. The objective of the Act
is to ensure inter-generational equity in fiscal management, long run macroeconomic stability, better
coordination between fiscal and monetary policy, and transparency in fiscal operation of the Government.
• FRBM Act provides a legal institutional framework for fiscal consolidation. It is now mandatory for the
Central government to take measures to reduce fiscal deficit, to eliminate revenue deficit and to generate
revenue surplus in the subsequent years. The Act binds not only the present government but also the
future Government to adhere to the path of fiscal consolidation.
• The Government can move away from the path of fiscal consolidation in certain cases as oresribed by the
law or other exceptional grounds which Central Government may specify.

4 www.visionias.in ©Vision IAS


• The subsection 4 (2) of the Act says about various grounds on which the FRBM’s fiscal deficit target may
be exempted during a year.
o National security, act of war
o National calamity
o Collapse of agriculture severely affecting farm output and incomes
o Structural reforms in the economy with unanticipated fiscal implications
o Decline in real output growth of a quarter by at least three per cent points below its average of the
previous four quarters
• Hence option (b) is the correct answer.

Q 13.A
• The International Association of Insurance Supervisors (IAIS) is a voluntary membership
organization of insurance supervisors from over 190 jurisdictions, constituting 97% of the world's
insurance premiums. It is the international standards-setting body for the insurance sector. The IAIS
was established in 1994 and operates as a Verein, a type of non-profit organization under Swiss Civil
Law.
• The IAIS' mission is to promote effective and globally consistent supervision of the insurance industry in
order to develop and maintain fair, safe and stable insurance markets for the benefit and protection of
policyholders and to contribute to the maintenance of global financial stability.
• Gujarat International Finance Tec (GIFT) City regulator International Financial Services Centres
Authority (IFSCA) has recently obtained membership of the International Association of Insurance
Supervisors (IAIS).
• Hence option (a) is the correct answer.

Q 14.B
• If you look at the graph given below no concrete relationship exists between employment and GDP
growth rate.
• Thus the GDP growth rate is neither directly nor inversely proportional to the employment growth
rate. Hence statement 1 is not correct.
• As is evident from the graph, the employment growth rate has always been lower than the rate of growth
of the GDP. The dark line has always been below the white line. Hence statement 2 is correct.
• The graph thus presents the story of the economic growth of India which has always been called 'jobless
growth'.

चमन Q¨V¨LV¢V
COURIER FACILITY ALL OVER INDIA
CONTECT NO. 9310098385,9551705170

Q 15.C
• Economic survey 2019-20 makes a comparative study with the labour market estimates based on Periodic
Labour Force Survey (PLFS) with the results of earlier quinquennial surveys on Employment-
Unemployment conducted by National Sample Survey Organization (NSSO).
5 www.visionias.in ©Vision IAS
• According to the Economic survey 2019-20:
o there has been an increase in the share of formal employment, as captured by ‘Regular wage/salaried’,
from 17.9 per cent in 2011-12 to 22.8 per cent in 2017-18.
o the proportion of women workers in ‘Regular wage/salaried’ employees category has increased
by 8 percentage points (from 13 per cent in 2011- 12 to 21 per cent in 2017-18) with the addition
of 0.7 crore new jobs for female workers in this category. Hence statement 1 is correct.
o there is drop-in casual labour who have mainly originated from the rural sector where rural
labourers have shifted from agricultural to industrial and services activity. Hence statement 2
is correct.
o In the urban region, there has been a shift in employment from self-employed to salaried jobs.

Q 16.B
• As per the Economic Survey 2019-20, the government has taken the following steps for the formalisation
of the Labour Market.

• Hence all the options are correct.

Q 17.C
• A base year is the first of a series of years in an economic or financial index. It is typically set to an
arbitrary level of 100. New, up-to-date base years are periodically introduced to keep data current in a
particular index. Any year can serve as a base year, but analysts typically choose recent years.
• A base year is used for comparison in the measure of a business activity or economic index. Hence,
statement 1 is correct.
• A revision in the base year is essential for better policymaking. It is meant to track structural changes in
an economy and improve or update macroeconomic indicators that reflect the economic performances of a
country. Hence, statement 2 is correct.
• Currently, the base year for GDP calculation is 2011-12 and the base year for CPI calculation is
2012. Hence, statement 3 is not correct.

Q 18.C
• Unemployment can be defined as a state of worklessness for a person who is fit and willing to work at the
current wage rate. It is a condition of involuntary and not voluntary idleness. Involuntary unemployment
can be further divided into cyclical unemployment, seasonal unemployment, structural unemployment,
frictional unemployment, the natural rate of unemployment, disguised unemployment and under-
employment.
6 www.visionias.in ©Vision IAS
• Seasonal unemployment occurs in a particular time of the year or season and thus is known as
seasonal unemployment. Seasonal unemployment is most common in industries like agriculture, tourism,
hotel, catering etc. Hence option 1 is not correct.
• Frictional unemployment occurs as a result of people voluntarily changing jobs within an
economy. After a person leaves a company, seeking a better job or being fired from a current job. Hence
option 2 is correct.
• Underemployment occurs when a person is employed but not in a befitting position or salary
corresponding to his qualification. Hence option 3 is correct.
• Structural unemployment comes about through a technological change in the structure of the
economy in which labour markets operate. It is a mismatch between the supply and demand for certain
skills in the labour market. For e.g. Technological changes such as the replacement of bicycle by
automobiles or the automation of manufacturing—lead to unemployment among workers displaced from
the bicycle sector. Hence option 4 is not correct.

Q 19.B
• The capital adequacy ratio is the percentage of total capital to the total risk-weighted assets.
• CAR, a measure of a bank’s capital, is expressed as a percentage of a bank’s risk-weighted.
• credit exposures:
CAR= Total of Tier 1 & Tier 2 capitals ÷ Risk-Weighted Assets.
• Also known as ‘Capital to Risk-Weighted Assets Ratio (CRAR)’ this ratio is used to protect depositors
and promote the stability and efficiency of financial systems around the world. Hence option (a) is not
correct.
• The capital adequacy ratio (CAR) norm has been the last provision to emerge in the area of regulating the
banks in such a way that they can sustain the probable risks and uncertainties of lending.
• The accord was agreed upon at Basel, Switzerland at a meeting of the Bank for International Settlements
(BIS). Hence option (b) is correct.
• As per RBI norms, Indian scheduled commercial banks are required to maintain a CAR of 9% while
Indian public sector banks are emphasized to maintain a CAR of 12%. Hence option (c) not correct.
• The RBI introduced the capital-to-risk weighted assets ratio (CRAR) system for the banks operating in
India in 1992 in accordance with the standards of the BIS—as part of the financial sector reforms. In the
coming years the Basel norms were extended to term-lending institutions, primary dealers and non-
banking financial companies (NBFCs), too. Hence option (d) is not correct.

Q 20.D
• The All-India Consumer Price Index Numbers for Agricultural Labourers and Rural Labourers (Base:
1986-87=100) is published by the Ministry of Labour & Employment on monthly basis. Hence statement
2 is not correct.
• There is demand going on to link wages under the rural employment guarantee scheme to consumer
price index-rural (CPI-RL). But currently, wage rates for workers under the Mahatma Gandhi
National Rural Employment Guarantee Act (MGNREGA), 2005 are notified and revised annually
based on Consumer Price Index-Agricultural Labourers (CPI-AL) by the Central Government in
accordance with the provisions of Section 6(1) of the MGNREGA Act, 2005. CPI-AL is published by
Ministry of Labour & Employment. Hence statement 1 is not correct.
• MGNREGA is a demand-driven wage employment programme. It provides livelihood security for rural
households. It envisages for providing 100 days of guaranteed wage employment in a financial year to
each household in rural areas whose adult members volunteers to do unskilled manual work. In addition to
this, there is a provision for additional 50 days of unskilled wage employment in a financial year in
drought/natural calamity notified rural areas.

Q 21.A
• Index of Industrial Production (IIP) measures the quantum of changes in industrial production in an
economy and captures the general level of industrial activity in the country. The current base year for
the IIP series in India is 2011-12.
• Categorization of IIP:
o Use-based classification of industries under Index of Industrial Production: In addition to the
industry-wise indices/growth rates, the users also require the indices in respect of different use-based
categories, i.e., basic goods, capital goods, intermediate goods, consumer durables and consumer non-
durables.
7 www.visionias.in ©Vision IAS
Weights of the different sectors under the used based classification – 2011-12 series

• Hence the correct sequence in decreasing order is: 1-2-3. Hence option (d) is the correct answer.
o Sectoral classification Industrial production for the purpose of IIP is divided into three sectors,
i.e, Mining, Manufacturing and Electricity. In 'Sectoral' classification, relative weights of
Manufacturing, Mining and Electricity are 75.5%, 14.2% and 10.3% respectively.

• Core industries in the IIP: The Eight Core Industries comprise 40.27 % of the weight of items included
in the Index of Industrial Production (IIP). These industries are: Coal, Crude Oil, Natural Gas, Refinery
Products, Fertilizers, Steel, Cement and Electricity.

8 www.visionias.in ©Vision IAS


Q 22.A
• Fiat money, in a broad sense, all kinds of money that are made legal tender by a government decree or
fiat. The term is, however, usually reserved for legal-tender paper money or coins that have face values far
exceeding their commodity values and are not redeemable in gold or silver. Hence statement 1 is
correct.
• Fiat currency or “real currency,” “real money,” or “national currency” is distinguished from Virtual
currency or digital currency. Fiat currency is the coin and paper money of a country that is designated as
its legal tender. While digital currency means a digital representation of either virtual currency (non-fiat)
or e-money (fiat) and thus is often used interchangeably with the term “virtual currency”. Hence
statement 2 is not correct.

Q 23.C
• The International Monetary Fund (IMF), headquartered in Washington, works to foster global
monetary cooperation, secure financial stability, facilitate international trade, promote high employment
and sustainable economic growth, and reduce poverty around the world.
• IMF is primarily funded through members' quotas.
o Each member of the IMF is assigned a quota (membership fee), part of which is payable in special
drawing rights (SDRs) or specified usable currencies ("reserve assets"), and part in the member's own
currency. The difference between a member's quota and the IMF's holdings of its currency is a
country's Reserve Tranche Position (RTP).
o RTP is accounted among a country's foreign-exchange reserves. Part of the quota can be withdrawn
from the IMF without any interest during critical situations of a country such as Balance of Payment
(BOP) crises. This part of the money which can be withdrawn without any interest is the RTP.
• Contingent Reserve Arrangement is a BRICS framework that aims at providing support through
additional liquidity and other means to BRICS countries at a time of economic crisis. It will be providing
support to BRICS countries through liquidity and precautionary instruments in response to actual or
potential short-term balance of payments pressures. The initial total committed resources of the CRA will
be 100 billion dollars with individual commitments as follows: China ($41 billion), India ($18 billion),
Brazil ($18 billion), Russia ($18 billion) and South Africa ($5 billion).
• Project Preparation Special Fund is a multidonor facility with the primary purpose of supporting
eligible Asian Infrastructure Investment Bank (AIIB) members, especially low-income members—
prepare bankable infrastructure projects AIIB may finance. It provides technical assistance grants for
preparing bankable infrastructure projects. Through these grants, clients can hire experts and consultants
to carry out the required preparation work. Project preparation activities supported by these grants are
critical to improve project quality, ensure bankability in terms of economic, environmental and social
sustainability and implementation readiness.
• Hence option (c) is the correct answer.

Q 24.B
• The Small Finance Bank (SFB) is a private financial institution intended to further the objective of
financial inclusion by primarily undertaking basic banking activities of acceptance of deposits and lending
to un-served and underserved sections including small business units, small and marginal farmers, micro
and small industries and unorganized sector entities, but without any restriction in the area of operations,
unlike Regional Rural Banks or Local Area Banks. Hence statement 1 is correct.
• The objectives of setting up of small finance banks were for furthering financial inclusion by (i) provision
of savings vehicles primarily to unserved and underserved sections of the population, and (ii) supply of
credit to small business units; small and marginal farmers; micro and small industries; and other
unorganized sector entities, through high technology-low cost operations. Hence statement 2 is correct.
• However, Small Financial Banks cannot be a business correspondent for another bank. Hence statement
3 is not correct.

Q 25.D
• Product Taxes and subsidies and Production Taxes and subsidies are used by CSO (now NSO) in the
calculation of GVA at basic prices and GVA at market prices. Gross value added (GVA) is an economic
productivity metric that measures the contribution of a corporate subsidiary, company or municipality to
an economy, producer, sector or region.
• Production taxes and subsidies are paid or received in relation to production and are independent of the
volume of production such as land revenues, stamp and registration fee. Hence, statement 2 is not
correct.
9 www.visionias.in ©Vision IAS
• Product taxes and subsidies, on the other hand, are paid or received per unit or product, e.g., excise tax,
service tax, export and import duties etc. Hence, statement 1 is not correct.
• GVA at factor costs + Net production taxes = GVA at basic prices + Net product taxes = GVA at market
prices

Q 26.A
• Interest rates are a monetary policy tool used by central banks to influence inflation throughout an
economy. In case of a negative interest rate a central bank attempts to combat deflation by reducing
interest rates past zero, into negative rates.
• In this situation instead of receiving money on deposits in the form of interest, depositors must pay
regularly to keep their money with the bank.
Impacts of Negative interest rates on the economy:
• lower borrowing costs as banks are willing to lend money more freely.
• Negative rates weaken currency which would encourage exports.
• Negative rates would discourage investment in domestic markets and investors tend to search for better
returns in foreign markets.
Q 27.D
• Gross capital formation is defined as outlays on additions to fixed assets, plus the net change in
inventories. Fixed assets include plant, machinery, equipment, and buildings, all used to create goods and
services. Inventory includes raw materials and goods available for sale. Hence statement 1 is correct.
• Countries need capital goods to replace the older ones that are used to produce goods and services. If a
country cannot replace capital goods as they reach the end of their useful lives, production declines.
Generally, the higher the capital formation of an economy, the faster an economy can grow its aggregate
income. So, if additional capital goods are utilized, it will lead to increase in goods and services. Hence
statement 2 is correct.
• GCF of India and neighbouring countries, according to World Bank Data
• India – 30 ( 2019)
• Pakistan – 16 (2019)
• Sri Lanka - 27 (2019)
• Bangladesh – 32 (2019)
• Nepal – 57 (2019)
• Bhutan – 47 (2018)
• Afghanistan – 18 (2019)
• As we can see that the GCF of India is more than Pakistan, Sri Lanka and Afghanistan. It is less than
Bangladesh, Nepal, Bhutan (2018 data).Hence statement 3 is correct.

Q 28.C
• In India, food prices rose steadily during the last months of 2009 and the early months of 2010, even
though the prices of non-food items continued to be relatively stable. As this somewhat unusual
phenomenon stubbornly persisted, and policymakers conferred on how to bring it to an end, the term
‘skewflation’ made an appearance in the Economic Survey 2009-10, Government of India, Ministry
of Finance.
• Given that it is sector specific, it is not evident that it calls for monetary or fiscal policy action. On the
other hand, given its sustained nature, it is not possible for government to ignore it, since causes stress to
consumers.
• Skewflation means the skewness of inflation among different sectors of the economy - some goods
and services are facing huge inflation, some none and some deflation. In other words, it means "when
there is a price rise of one or a small group of commodities over a sustained period of time while
Inflation in the remaining goods and services remain usual".
• Hence option (c) is the correct answer.

Q 29.D
• House price is an essential indicator of the economy. It has a co-movement with the Gross Domestic
Product (GDP), consumption, investment, inflation and current account balance. The variations in
residential property price affect households’ long-term investment strategy influence their spending and
borrowing patterns.
• In view of this, residential property price is an important piece of information for policymakers, especially
for the central banks. In the Indian context, there are two established housing price indices :
10 www.visionias.in ©Vision IAS
o House Price Index (HPI) compiled by the Reserve Bank of India and
o RESIDEX, which is compiled by the National Housing Bank (NHB).
• Therefore, Reserve Bank publishes house price index (HPI) quarterly (base: 2010-11=100) based on
transaction level data received from housing registration authorities in ten major cities (viz., Ahmedabad,
Bengaluru, Chennai, Delhi, Jaipur, Kanpur, Kochi, Kolkata, Lucknow and Mumbai).
• Hence option (d) is the correct answer.

Q 30.B
• Statement 1 is correct and statement 2 is not correct: Recently, The Union Environment Ministry has
constituted a high-level inter-ministerial Apex Committee for Implementation of Paris Agreement
(AIPA) under the chairmanship of Secretary, Ministry of Environment, Forest and Climate Change
(MoEFCC) to ensure that India was “on track” towards meeting its obligations under the Paris
Agreement including its Nationally Determined Contributions (NDC).
• Set up under sub-section (3) of section 3 of the Environment (Protection) Act, 1986 (29 of 1986), the
Apex Committee will function as a National Authority to regulate the “Carbon Markets” in India under
Article 6.2, Article 6.4 and Article 6.8 of the Paris Agreement in the post-2020 period.
• India submitted its NDC in 2015 for implementation of the Paris Agreement in the post-2020 period. The
NDC has eight goals including three quantitative goals namely a reduction in the emissions intensity of
Gross Domestic Product (GDP) by 33 to 35% by 2030 from the 2005 level; achieving about 40%
cumulative electric power installed capacity from non-fossil fuel-based energy resources by 2030; and
creating an additional carbon sink of 2.5 to 3 billion tonnes of carbon dioxide equivalent through
additional forest and tree cover by 2030.
• Statement 3 is correct: Another key function of the AIPA would be to operate as a National
Authority to regulate carbon markets in India under Article 6 of the Paris Agreement, formulate
guidelines for the consideration of projects or activities under Article 6 of the Paris Agreement, issue
guidelines on carbon pricing, market mechanism, and other similar instruments that have a bearing on
climate change and NDCs. It will take note of the contributions of the private sector and multi-/bi-lateral
agencies in the field of climate change and provide guidance for aligning their climate actions with
national priorities.

Q 31.B
• Stagflation is a situation in an economy when inflation and unemployment both are at higher levels,
contrary to conventional belief.
• Reflation is a situation often deliberately brought by the government to reduce unemployment and
increase demand by going for higher levels of economic growth. Governments go or higher public
expenditures, tax cuts, interest rate cuts, etc. Fiscal deficit rises, extra money is generally printed at a
higher level of growth, wages increase and there is almost no improvement in unemployment. Reflation
can also be understood from a different angle—when the economy is crossing a cycle of recession (low
inflation, high unemployment, low demand, etc.) and the government takes some economic policy
decisions to revive the economy from recession, certain goods see a sudden and temporary increase in
their prices, such price rise is also known as reflation.
• Skewflation there is a price rise of one or a small group of commodities over a sustained period of time,
without a traditional designation.
• Hence option (b) is the correct answer.

11 www.visionias.in ©Vision IAS


Q 32.D

Q 33.C
• The Medium-Term Fiscal Policy Statement cum Fiscal Policy Strategy Statement is presented to
Parliament under Section 3 of the Fiscal Responsibility and Budget Management Act, 2003.
• It sets out the three-year rolling targets for six specific fiscal indicators in relation to GDP at market
prices, namely
o Fiscal Deficit
o Revenue Deficit
o Primary Deficit
o Tax Revenue
o Non-tax Revenue
o Central Government Debt.
• The Statement includes the underlying assumptions, an assessment of the balance between revenue
receipts and revenue expenditure and the use of capital receipts including market borrowings for the
creation of productive assets. It also outlines for the existing financial year, the strategic priorities of the
Government relating to taxation, expenditure, lending and investments, administered pricing, borrowings
and guarantees.
• The Statement explains how the current fiscal policies are in conformity with sound fiscal management
principles and gives the rationale for any major deviation in key fiscal measures.
• Hence option (c) is the correct answer.

12 www.visionias.in ©Vision IAS


Q 34.B
• Fiscal drag is an economic term whereby inflation or income growth moves taxpayers into higher
tax brackets. This in effect increases government tax revenue without actually increasing tax rates. The
increase in taxes reduces aggregate demand and consumer spending from taxpayers as a larger share of
their income now goes to taxes, which leads to deflationary policies, or drag, on the economy.
• Fiscal drag is essentially a slowing in the growth of the economy caused by a lack of spending as
increased taxation slows the demand for goods and services. When an economy is rapidly expanding,
inflation results in higher income and therefore individuals moving into higher tax brackets and
paying more of their income in taxes. This is particularly the case in economies with progressive taxes,
or tax brackets, which stipulate that the higher income an individual makes the higher the tax they pay and
thus they move into a higher tax bracket.
• Moving into a higher tax bracket and paying a larger portion of income in taxes, as mentioned prior,
results in an eventual slowing of the economy as there is now less income available for discretionary
spending.
• It is common to view fiscal drag as a natural economic stabilizer as it tends to keep demand stable and the
economy from overheating. This is generally viewed as a mild deflationary policy and a positive aspect to
fiscal drag.
• Hence option (b) is the correct answer.

Q 35.C
• If the expenditure of the government exceeds its income, the government is said to have incurred a fiscal
deficit. This deficit financing has to be done either by borrowing from the market or monetisation of
deficit through RBI.
• Monetisation of Deficit: It involves the financing of such extra expenses with money, instead of debt
to be repaid at some future dates. So, it is a form of non-debt financing. As a result, under
monetization, there is no increase in net (not gross) public debt. Hence statement 1 and 2 are not
correct. It can occur only through one of two modalities:
o Direct Monetization
o Indirect Monetization
• Direct Monetization (DM): Under this method, RBI prints new currency and purchases government
bonds directly from the primary market (from the government) using this currency. As a result, this
supports the spending needs of the government.
• Indirect monetization (IM): In this method, deficits are monetized as the government issues bonds in the
primary market and the RBI purchases an equivalent amount of government bonds from the secondary
market in the form of Open Market Operations (OMO).
• Monetisation of deficit was in practice in India till 1997, whereby the central bank automatically
monetised government deficit through the issuance of ad-hoc treasury bills.
• In 1994 and 1997, two agreements were signed between the government and RBI to completely phase out
funding through ad-hoc treasury bills.
• Later on, with the enactment of Fiscal Responsibility and Budget Management (FRBM) Act, 2003,
RBI was completely barred from subscribing to the primary issuances of the government. It was
agreed that henceforth, the RBI would operate only in the secondary market through the OMO
(open market operations) route. Hence statement 3 is correct.

Q 36.B
• Assets are things a firm owns or what a firm can claim from others. In case of a bank, apart from
buildings, furniture, etc., its assets are loans given to the public. When the bank gives out loan of Rs 100
to a person, this is the bank’s claim on that person for Rs 100. Another asset that a bank has reserves.
Reserves are deposits which commercial banks keep with the Central bank, Reserve Bank of India (RBI)
and its cash. These reserves are kept partly as cash and partly in the form of financial instruments (bonds
and treasury bills) issued by the RBI. Reserves are similar to deposits we keep with banks. We keep
deposits and these deposits are our assets, they can be withdrawn by us. Similarly, commercial banks like
State Bank of India (SBI) keep their deposits with RBI and these are called Reserves.
• Assets = Reserves + Loans.

13 www.visionias.in ©Vision IAS


• Liabilities for any firm are its debts or what it owes to others. For a bank, the main liability is the deposits
which people keep with it.
• International Assets of Banks in India - By Type of Instruments: 1. Loans and Deposits of which- Loans
to Non-residents, Foreign Currency Loan to Residents, Outstanding Export Bills, Foreign Currency in
hand, Travellers Cheques, etc., NOSTRO Balances and Placements Abroad. 2. Holdings of Debt
Securities 3. Other International Assets. Hence option (b) is correct answer.

Q 37.C
• Recent Context: Recently, a farmers' association in Kerala sent a truckload of pineapples to Delhi for
free distribution among the farmers protesting at the borders of NCR.
• Trucks, loaded with 16 tonnes of pineapples, were sent from Vazhakulam - a town in Kerala's
Ernakulam District, popularly referred to as the 'Pineapple City'.
o For the uninitiated, Vazhakulam pineapple is deemed to be the sweetest pineapple in India and
received the GI tag in 2009.
o It has high-sugar content and low acidity.

Q 38.D
• As per the data released by the National Statistical Office (NSO), retail inflation jumped to an eight-
month high of 7.34 per cent in September 2020, primarily driven by higher food inflation.
• The 7.34 per cent, the inflation print is above the Reserve Bank of India’s medium-term target of 4+/- 2
per cent.
• As food inflation adversely impact the poor relatively more, in order to counter it, the Government
has taken various measures from time to time to stabilize prices of essential food items which,
include appropriately utilizing trade and fiscal policy instruments like import duty, Minimum
Export Price, export restrictions, etc. to regulate domestic availability and moderate prices. Hence
option is 1 correct.
o Minimum Export Price (MEP) is the price below which an exporter is not allowed to export the
commodity from India. MEP is imposed in view of the rising domestic retail / wholesale price or
production disruptions in the country. It is a kind of quantitative restriction to trade.
Government fixes MEP for the selected commodities with a view to arrest domestic price rise
and augment domestic supply. This is intended to be imposed for short durations and is removed
when situations change.
o As per section 5 of The Foreign Trade (Development And Regulation) Act, 1992, the Central
Government may, from time to time, formulate and announce by notification in the Official Gazette,
the export and import policy and may also, in the like manner, amend that policy. Notifications on
MEP issued by Department of Commerce.
• Buffer stock refers to a reserve of a commodity that is used to offset price fluctuations and
unforeseen emergencies. Buffer stock is generally maintained for essential commodities and
necessities like food grains, pulses etc. Buffer Stock Policy of the Government of India (GOI) The
concept of buffer stock was first introduced during the IVth Five Year Plan (1969-74). A buffer stock of
food grains in the Central Pool is maintained by the Government of India (GOI) / Central Government for:
o Meeting the prescribed minimum buffer stock norms for food security,
o The monthly release of food grains for supply through the Targeted Public Distribution System
(TPDS) and Other Welfare Schemes (OWS),
o Meeting emergency situations arising out of unexpected crop failure, natural disasters, etc.,
o Price stabilisation or market intervention to augment supply so as to help moderate the open
market prices. Hence option 2 is correct.
o The Cabinet Committee on Economic Affairs fixes the minimum buffer norms on a quarterly basis.
• Food Park aims at providing a mechanism to link agricultural production to the market by bringing
together farmers, processors and retailers so as to ensure maximizing value addition, minimizing
wastage. Providing credit-linked subsidies will encourage people to invest in Mega food parks with
the support of government and it will result in better infrastructure and reduction in agricultural
waste. It will enhance the supply of agriculture commodities and helpful in countering food
inflation. Hence option 3 is correct.

Q 39.D
• AePS is a bank led model which allows online interoperable financial inclusion transaction at PoS
(MicroATM) through the Business correspondent of any bank using the Aadhaar authentication. It will
further speed track Financial Inclusion in the country. Hence statement 1 is correct.
14 www.visionias.in ©Vision IAS
• As more people are coming under the ambit of AEPS so the process of digital transactions is increasing
day by day. As a result, one can witness a pivot to reliance on the Aadhaar enabled Payment System
(AePS), which is emerging as a key cash-out medium. This would imply that the use of ATMs would
reduce considerably. Hence statement 2 is correct.
• Few other benefits of AEPS are:
o It is safe and secure payment method.
o It is Interoperable across various banks.
o Through AEPS, all bank account holders will be able to access their bank accounts through Aadhaar
authentication .
o With AEPS, the only information required to initiate a transaction is Aadhaar number and biometric
information. Hence statement 3 is correct.

Q 40.D
• Recent Context: The New Development Bank (NDB) will lend India $1 billion for supporting the
country’s economic recovery from the Covid-19 pandemic through expenditure on rural infrastructure
related to natural resource management.
o The funds will also be utilised for employment generation under the Mahatma Gandhi National Rural
Employment Guarantee Scheme.
o The loan has a repayment tenor of 30 years, including a five-year grace period.
• The New Development Bank (NDB), formerly referred to as the BRICS Development Bank, is a
multilateral development bank established by the BRICS states (Brazil, Russia, India, China and
South Africa). Hence option (b) is the correct answer.
• According to the Agreement on the NDB, "the Bank shall support public or private projects through loans,
guarantees, equity participation and other financial instruments." Moreover, the NDB "shall cooperate
with international organizations and other financial entities, and provide technical assistance for projects
to be supported by the Bank."
• The initial authorized capital of the bank is $100 billion divided into 1 million shares having a par
value of $100,000 each. The initial subscribed capital of the NDB is $50 billion divided into paid-in
shares ($10 billion) and callable shares ($40 billion).
• The initial subscribed capital of the bank was equally distributed among the founding
members. The Agreement on the NDB specifies that every member will have one vote no one would
have any veto powers.
• NDB is currently headquartered in BRICS Tower (former Oriental Financial Centre) in Shanghai The
bank is headquartered in Shanghai, China. The first regional office of the NDB is in Johannesburg,
South Africa.

Q 41.B
• Recent Context: India chaired the 19th meeting of the Council of Heads of Governments (CHG), i.e.,
Prime Ministers, of the Shanghai Cooperation Organisation (SCO) in a virtual format.
o This was the first summit meeting of the SCO to be hosted by India since it became its member in
2017.
• The Shanghai Cooperation Organisation (SCO), or Shanghai Pact, is a Eurasian political,
economic, and security alliance, the creation of which was announced on 15 June 2001 in Shanghai,
China by the leaders of China, Kazakhstan, Kyrgyzstan, Russia, Tajikistan, and Uzbekistan; the
Shanghai Cooperation Organisation Charter, formally establishing the organisation, was signed in June
2002 and entered into force on 19 September 2003.
• The original five nations, with the exclusion of Uzbekistan, were previously members of
the Shanghai Five group, founded on 26 April 1996. Since then, the organisation has expanded its
membership to eight countries when India and Pakistan joined SCO as full members on 9 June
2017 at a summit in Astana, Kazakhstan. Hence the correct option is (b)
• The Heads of State Council (HSC) is the supreme decision-making body in the SCO, it meets once a
year and adopts decisions and guidelines on all important matters of the organisation. Military exercises
are also regularly conducted among members to promote cooperation and coordination against terrorism
and other external threats, and to maintain regional peace and stability.
• The SCO is the largest regional organisation in the world in terms of geographical coverage and
population, covering three-fifths of the Eurasian continent and nearly half of the human population.
• Criticisms of the SCO include that it is used by member states to shield each other from international
criticism regarding human rights violations.

15 www.visionias.in ©Vision IAS


Q 42.D
• Gross domestic product (GDP) is the total monetary or market value of all the finished goods and services
produced within a country's borders in a specific time period.
• Irrespective of who owns the company or firm, if the goods or service is produced within the geographical
boundaries of a country, it will be counted in the GDP of the country else not.
• Since cars A and C are manufactured in India, they will be counted in the GDP of India. But car B which
is manufactured in Britain will be counted under the GDP of Britain and not that of India. Hence option
(d) is the correct answer.

Q 43.C
• Recent Context: The FATF (Financial Action Task Force), in collaboration with Egmont Group, has
released a new report to help the public and private sector with the challenges of detecting trade-based
money laundering (TBML).
• The Financial Action Task Force (on Money Laundering) (FATF), also known by its French name,
Groupe d'action financière (GAFI), is an intergovernmental organisation founded in 1989 on the initiative
of the G7 to develop policies to combat money laundering. In 2001, its mandate was expanded to include
terrorism financing.
• The objectives of FATF are to set standards and promote effective implementation of legal, regulatory
and operational measures for combating money laundering, terrorist financing and other related threats to
the integrity of the international financial system. FATF is a "policy-making body" that works to generate
the necessary political will to bring about national legislative and regulatory reforms in these areas. FATF
monitors progress in implementing its Recommendations through "peer reviews" ("mutual evaluations")
of member countries.
• FATF Blacklists
o Officially known as High-Risk Jurisdictions subject to a Call for Action, the FATF blacklist sets
out the countries that are considered deficient in their anti-money laundering and counter financing of
terrorism regulatory regimes. The list is intended to serve not only as a way of negatively highlighting
these countries on the world stage, but as a warning of the high money laundering and terror financing
risk that they present. It is extremely likely that blacklisted countries will be subject to economic
sanctions and other prohibitive measures by FATF member states and other international
organizations. North Korea and Iran are the only two blacklisted countries.
• FATF Greylists
o In addition to its blacklist, the FATF also issues a grey list, officially referred to as Jurisdictions
Under Increased Monitoring. Like the blacklist, countries on the FATF grey list represent a much
higher risk of money laundering and terrorism financing but have formally committed to working
with the FATF to develop action plans that will address their AML/CFT deficiencies.
o Countries on the list may face economic sanctions from institutions like the IMF and the World Bank
and experience adverse effects on trade. Pakistan is on the Greylist.
• Hence option (c) is the correct answer.

Q 44.B
• Headline Inflation reflects the rate of change in prices of all goods and services in an economy over a
period of time. Whereas Core Inflation is a measure of inflation which excludes items that face volatile
price movement, notably food and energy. Core Inflation is nothing but Headline Inflation minus inflation
that is contributed by food and energy commodities. Hence, statement 1 is not correct.
• Food and fuel prices are relatively more volatile due to various factors affecting the agriculture sector or
the oil economy. On the other hand, prices of other commodities do not fluctuate as regularly as food and
fuel – as such increase in their prices could be taken relatively to be much more of a permanent nature. If
this is so, then it follows logically for Central Banks to target only core inflation, as it reflects the demand-
side pressure in the economy.
• Whenever core inflation rises, Central Banks increase their key policy rates to suck excess liquidity from
the market and vice versa. It is, therefore, a preferred tool for framing long-term policy. Responding
to headline inflation might therefore sometimes be inappropriate as it generates excessive variability in the
unemployment rate – variability that would be much more subdued when policy responds to core
inflation. Hence, statement 2 is correct.

Q 45.D
• Liquidity trap is a situation when expansionary monetary policy (increase in the money supply) does not
increase the interest rate, income and hence does not stimulate economic growth.
16 www.visionias.in ©Vision IAS
• Liquidity trap is the extreme effect of monetary policy.
• It is a situation in which the general public is prepared to hold on to whatever amount of money is
supplied, at a given rate of interest.
• They do so because of the fear of adverse events like deflation, war.
• In that case, a monetary policy carried out through open market operations has no effect on either the
interest rate or the level of income.
• In a liquidity trap, the monetary policy is powerless to affect the interest rate.
• Hence option (d) is the correct answer.

Q 46.D
• An inflationary gap is a macroeconomic concept that measures the difference between the current level of
real GDP and the gross domestic product (GDP) that would exist if an economy was operating at full
employment.
• For the gap to be considered inflationary, the current real GDP must be higher than the potential GDP.
The inflationary gap exists when the demand for goods and services exceeds production due to factors
such as higher levels of overall employment, increased trade activities or increased government
expenditure.
• Therefore, increasing the salaries of government employees and enhancing the minimum support
price (MSP) of the notified agricultural commodities will lead to an increase in government
expenditure. It will create more demand in the economy and further widen the inflationary gap.
Hence options 1 and 2 are not correct.
• However, following policies or decisions can be taken by the government to reduce inflationary gap such
as reductions in government spending, increase in the tax rate, increasing interest rate increases, transfer
payment reductions and issuing bond and securities.
• Dated G-Secs are long-term securities which carry a fixed or floating coupon (interest rate) which is paid
on the face value, on a half-yearly basis. Generally, the tenor of dated securities ranges from 5 years to 40
years. Issuing fresh dated securing will reduce the available money and demand in the market
consequently. Therefore It will lead to a reduction in an inflationary gap. Hence option 3 is correct.
• Additional Information: A Government Security (G-Sec) is a tradeable instrument issued by the Central
Government or the State Governments. It acknowledges the Government’s debt obligation. Such
securities are short term (e.g. treasury bills (T-Bill) or long term (usually called Government bonds or
dated securities with an original maturity of one year or more). G-Secs carry practically no risk of default
and hence, are called risk-free gilt-edged instruments.

Q 47.D
• Recent Context: India is considering a plan to build a 10 gigawatts (GW) hydropower project in a
remote eastern state on the Brahmaputra.
• The Brahmaputra called the Yarlung Tsangpo in Tibet, with its origin in the Manasarovar Lake
region, near Mount Kailash. Siang/Dihang River in Arunachal Pradesh and Luit, Dilao in Assam, is a
trans-boundary river that flows through China, India, and Bangladesh. It is the 9th largest river in the
world by discharge and the 15th longest.
• The Brahmaputra is of great importance as it accounts for 29% of the total runoff of India's rivers and
its basin possesses about 44% of India's total hydropower potential.
• Hence option (d) is the correct answer.

Q 48.A
• A J-curve is a trendline that shows an initial loss immediately followed by a dramatic gain.
• In a chart, this pattern of activity would follow the shape of a capital "J".
• The J-curve effect is often cited in economics to describe, for instance, the way that a country’s balance
of trade initially worsens following a devaluation of its currency, then quickly recovers and finally
surpasses its previous performance.
• In economics, the J-curve shows how a currency depreciation causes a severe worsening of a trade
imbalance followed by a substantial improvement. Hence, option (a) is the correct answer.
• The pattern is as follows:
o Immediately after a nation's currency is devalued, imports get more expensive and exports get
cheaper, creating a worsening trade deficit (or at least a smaller trade surplus).
o Shortly thereafter, the sales volume of the nation's exports begins to rise steadily, thanks to their
relatively cheap prices.
o At the same time, consumers at home begin to buy more locally-produced goods because they are
relatively affordable compared to imports.
17 www.visionias.in ©Vision IAS
o Over time, the trade balance between the nation and its partners bounces back and even exceeds pre-
devaluation times.

Q 49.A
• An Emergency Use Authorization (EUA) is a mechanism to facilitate the availability and use of medical
counter-measures, including vaccines, during public health emergencies, such as the current COVID-19
pandemic.
o Under an EUA, regulator may allow the use of unapproved medical products, or unapproved
uses of approved medical products in an emergency to diagnose, treat, or prevent serious or life-
threatening diseases or conditions when certain statutory criteria have been met, including that
there are no adequate, approved, and available alternatives till the completion of final trials.
Hence statement 1 is correct.
o Thus, in emergency situations, like the current one, regulatory authorities around the world have
developed mechanisms to grant interim approvals if there is sufficient evidence to suggest a medical
product is safe and effective. Final approval is granted only after completion of the trials and analysis
of full data; until then, emergency use authorization (EUA) allows the medicine or the vaccine to be
used on the public.
o In fact, approval from the regulator is required at every stage of these trials. This is a long process,
designed to ensure that medicine or vaccine is absolutely safe and effective. The fastest approval for
any vaccine until now — the mumps vaccine in the 1960s — took about four-and-a-half years after it
was developed.
o Vaccines and medicines, and even diagnostic tests and medical devices, require the approval of a
regulatory authority before they can be administered.
• In India, the regulatory authority for such mechanism is the Central Drugs Standard Control
Organisation (CDSCO).
• India doesnot have a phrase called EUA in any of its laws, but there are provisions under the New
Drugs and Clinical Trial Rules, 2019, to grant such permissions. Hence statement 2 is not correct.
• Three vaccine manufacturers — SII, Pfizer Inc and Bharat Biotech Intl Ltd — have already applied for
emergency authorisations to the Drugs Controller General of India (DCGI). While SII and Pfizer have
presented a preliminary analysis of their phase-3 trial, Bharat Biotech is yet to start it.

Q 50.D
• The Statutory Liquidity Ratio (SLR) is a prudential measure under which (as per the Banking Regulations
Act 1949) all Scheduled Commercial Banks in India must maintain an amount in one of the following
forms as a percentage of their total Demand and Time Liabilities (DTL) / Net DTL (NDTL);
[i] Cash.
[ii] Gold; or
[iii] Investments in un-encumbered Instruments that include;
(a) Treasury-Bills of the Government of India.
(b) Dated securities including those issued by the Government of India from time to time under the market
borrowings programme and the Market Stabilization Scheme (MSS).
(c) State Development Loans (SDLs) issued by State Governments under their market borrowings
programme.
(d) Other instruments as notified by the RBI.
• Hence option (d) is correct answer.

Q 51.A
• Recently, the International Labour Organisation (ILO) has released the World Employment and
Social Outlook: Trends 2020 (WESO) report
• World Employment and Social Outlook: Trends 2020. Hence option (a) is the correct answer.
• This report provides an overview of global and regional trends in employment, unemployment,
labour force participation and productivity, as well as dimensions of job quality such as employment
status, informal employment and working poverty. It also examines income and social developments and
provides an indicator of social unrest. According to the new report, almost, half a billion people are
working fewer paid hours than they would like or lack adequate access to paid work.
• In addition, WESO shows that unemployment is projected to increase by around 2.5 million in 2020.
• The report shows that the mismatch between labour supply and demand extends beyond
unemployment into broader labour underutilization. In addition to the global number of unemployed (188
18 www.visionias.in ©Vision IAS
million), 165 million people don’t have enough paid work and 120 million have either given up actively
searching for work or otherwise lack access to the labour market. In total, more than 470 million people
worldwide are affected.
• The report also looks at the labour market inequalities. Using new data and estimates it shows that, at the
global level, income inequality is higher than previously thought, especially in developing countries.
• Moderate or extreme working poverty is expected to edge up in 2020-21 in developing countries,
increasing the obstacles to achieving Sustainable Development Goal 1 on eradicating poverty
everywhere by 2030.
• Currently working poverty (defined as earning less than US$3.20 per day in purchasing power parity
terms) affects more than 630 million workers or one in five of the global working population.
• Other flagship publications of ILO are:
o Global Wage Report which analyses the latest wage statistics and provides insights on wage policies
based on research projects carried out by the ILO.
o The World Social Protection Report which takes a comprehensive look at how countries are
investing in social security, how they are financing it, and how effective their approaches are.

Q 52.B
• Foreign exchange reserves are assets held on reserve by a central bank in foreign currencies. These may
include foreign currencies, bonds, treasury bills, and other government securities.
• Foreign exchange reserves of RBI include – Foreign Currency Assets, Gold, SDR (Special Drawing
Right), Reserve Position in the IMF. As of 20 November 2020, RBI’s forex reserves stood at US$
575.290.
• Forex reserves increase when there is an increased inflow of foreign currencies. Increase in exports and an
increase in inbound international tourism will increase the inflow of foreign currencies. Hence options (a)
and (c ) will lead to an increase in forex reserves of a country.
• Foreign Direct Investment (FDI) is an investment in the equities of companies by foreign entities. Increase
in FDI will cause an increased inflow of foreign currencies. This, in turn, will increase in forex
reserves. Hence, option (d) will also lead to an increase in forex reserves.
• However, if the world trade increases, it may or may not favour exports from a country. Hence, option
(b) may not necessarily lead to increase in forex reserves.

Q 53.B
• Part of the National Income (NI) which is received by the household is called Personal Income (PI). Out
of NI, which is earned by the firms and government enterprises, a part of the profit is not distributed
among the factors of production. This is called Undistributed Profits (UP). We have to deduct UP from NI
to arrive at PI, since UP does not accrue to the households. Similarly, Corporate Tax, which is imposed on
the earnings made by the firms, will also have to be deducted from the NI, since it does not accrue to the
households. On the other hand, the households do receive interest payments from private firms or the
government on past loans advanced by them. And households may have to pay interests to the firms and
the government as well, in case they had borrowed money from either. So, we have to deduct the net
interests paid by the households to the firms and government. The households receive transfer payments
from government and firms (pensions, scholarship, prizes, for example) which have to be added to
calculate the Personal Income of the households.
• Thus, Personal Income (PI) = NI – Undistributed profits – Net interest payments made by households –
Corporate tax + Transfer payments to the households from the government and firms.
• Even PI is not the income over which the households have complete say. They have to pay taxes from PI.
If we deduct the Personal Tax Payments (income tax, for example) and Non-tax Payments (such as fines)
from PI, we obtain what is known as the Personal Disposable Income.
• Personal Disposable Income (PDI) = PI – Personal tax payments – Non-tax payments. It can be seen
that value of Personal Disposable Income is less than the Personal Income. Hence statement 1 is not
correct.
• Personal Disposable Income is the part of the aggregate income which belongs to the households.
Hence statement 2 is correct. They may decide to consume a part of it and save the rest.

19 www.visionias.in ©Vision IAS


Q 54.C
• Recent Context: A recent report by Centre for Science and Environment (CSE) showed that only three
honey brands - Saffola, MarkfedSohna and Nature's Nectar - out of 13 passed the stringent Nuclear
Magnetic Resonance (NMR) test for testing the purity of their product.
o It claimed that household names including Dabur, Patanjali, Baidyanath, Zandu, Hitkari and Apis
were adulterating their honey with a modified sugar imported from China, which can bypass domestic
standard tests.
• Statement 1 is correct: Nuclear Magnetic Resonance (NMR) spectroscopy is an analytical chemistry
technique used in quality control and research for determining the content and purity of a sample
as well as its molecular structure. For example, NMR can quantitatively analyze mixtures containing
known compounds.
o When molecules are placed in a strong magnetic field, the nuclei of some atoms will begin to behave
like small magnets. If a broad spectrum of radiofrequency waves are applied to the sample, the nuclei
will be to resonate at their own specific frequencies. This is similar to the use of a tuning fork, where
a guitar string will only resonate in response to a tone of the exact right frequency.
o This technique allows for non-destructive screening and quantification of both known ingredients and
unanticipated contaminants and adulterants. Both targeted and untargeted NMR methodologies are
widely recognized as important tools for food authentication and the detection of counterfeit
products.
• Statement 2 is correct: Nuclear magnetic resonance (NMR) spectroscopy is a method that is gaining
ground in food analysis and has been successful in characterizing products such as beer, juice and
infant formulas, milk, etc. It has the advantages of being able to reveal multiple components of a food-
stuff on a single spectrum and, with the help of automation, can deliver rapid throughput analysis.
• The internationally accepted Nuclear Magnetic Resonance Spectroscopy (NMR) test to detect sugar
syrups in honey has limited utility in India. Globally, NMR is said to be the gold standard for testing
honey. The prior existence of a database is a necessity for the effective utilization of this technique. No
such database exists at present for Indian honey and hence, NMR testing has limited utility.

Q 55.B
• Private final consumption expenditure (PFCE) includes the final consumption expenditure of
o households and
o non-profit institutions serving households (NPISH) like temples, gurdwaras on the final consumption
of goods and services, whether made within or outside the economic territory. Hence option (b) is
the correct answer.
• The final consumption expenditure of households relates to outlays on new durable as well as
nondurable goods (except land) and on services.
o This consumption expenditure also includes the imputed gross rent of owner-occupied dwellings,
consumption of Own-account production evaluated at producers; prices and payments in kind of
wages and salaries valued at cost, e.g., provision for food, shelter and clothing to the employees.
• The final consumption expenditure of nonprofit institutions serving households includes the value of
goods and services produced for own use on current account i.e., the value of gross output reduced by the
sum of the value of their commodity and non-commodity sales.
o This would include transfers in kind of nondurable goods and services from Government
Administration, industries and rest of the world.
• The final consumption expenditure of households and nonprofit institutions serving households are
estimated together and are not available separately. However, expenditures on secondhand goods are
not included.
• Estimates of PFCE are obtained by following the commodity flow method.
o The commodity -flow method of estimating PFCE on a given item group consists of subtracting
intermediate consumption, Government final consumption, exports, changes in stocks and investment
from net availability given by domestic production plus imports.
o The quantity of final consumption obtained from commodity balances is generally evaluated at market
prices.
o PFCE is the biggest component in the calculation of GDP in the expenditure approach.

Q 56.D
• Fiscal Deficit is the difference between the total income of the government (total taxes and non-debt
capital receipts) and its total expenditure.
• A recurring high fiscal deficit means that the government has been spending beyond its means.
20 www.visionias.in ©Vision IAS
• Fiscal Deficit = Total expenditure – (Total receipts - debt creating receipts)
• In the wake of the recent COVID Crisis and looming economic slowdown, the government took various
initiatives to boost the economy and support the people. Some of the steps include Loan Moratorium,
Monetary Support to poor people through, Investment in vaccine research, borrowing from World Bank
for boosting health infrastructure etc.
o Loan Moratorium: The Centre approved the scheme for ‘grant of ex-gratia payment of the
difference between compound and simple interest to borrowers of specified loan accounts’ from
March 1-August 31. The ‘ex-gratia payment’ or the benefit will have to be routed through lending
institutions. The government would reimburse the claim amount to the lenders. The benefit will be
extended for loans availed across eight categories: micro, small and medium enterprises (MSMEs),
education, housing, consumer durables, credit card dues, automobile, personal and professional and
consumption. The scheme is valid for borrowers who availed the moratorium fully, partially or not at
all, as mentioned in the circular.
o Support to poor people: Bank account holders under Pradhan Mantri Jan Dhan Yojana (PMJDY)
were provided a monthly sum of Rs 500 during the lockdown period under the Pradhan Mantri Garib
Kalyan Yojana.
o Investment in vaccine research: Government announced Rs 900 crore funding to the Department of
Biotechnology for COVID-19 vaccine research. It is part of the Rs 2.65 lakh crore Atmanirbhar
Bharat 3.0 stimulus.
• The above-mentioned steps are an additional expenditure from the state exchequer and tend to widen the
fiscal deficit.
• Hence option (d) is the correct answer.

Q 57.C
• GDP is commonly used as the metric for evaluating the state of economic activity. In India, however,
there is an overwhelming preference of statisticians, the academia and policymakers for the use of gross
value added (GVA) in view of the latter’s superior sectoral coverage and tractability in terms of
explaining primary, secondary and tertiary areas of activity that together constitute aggregate supply.
While GVA gives a picture of the state of economic activity from the producers' side or supply side, the
GDP gives the picture from the consumers' side or demand perspective. Both measures need not match
because of the difference in treatment of net taxes. Hence, statement 1 is correct.
• GVA is also a better reflection of the productivity of producers as it excludes indirect taxes which could
distort an analytical view of the production process. As a result, high-frequency indicators of activity have
developed around the constituents of GVA and less robustly around GDP. In the same vein, however, it
can be argued that GDP is distorted by the presence of subsidies. Hence statement 2 is correct.
• From a practitioner’s perspective, an important factor hindering the use of GDP in India is the size of
Statistical Discrepancies (errors) embedded in it, relative to advanced economies and peer emerging
economies as well. Until recently too, Statistical Discrepancies in Indian national accounts have also
proved to be volatile - and more so in terms of constant prices – which disfigures an accurate assessment
of the underlying macroeconomic situation. In fact, GDP excluding Statistical Discrepancies compares
favourably with GVA in terms of means and volatility.

Q 58.C
• The Main Functions of the Reserve Bank of India is are as follows:
• Monetary Authority:
o Formulates, implements and monitors the monetary policy.
o Objective: maintaining price stability while keeping in mind the objective of growth.
• Regulator and supervisor of the financial system:
o Prescribes broad parameters of banking operations within which the country's banking and financial
system functions.
o Objective: maintain public confidence in the system, protect depositors' interest and provide cost-
effective banking services to the public.
• Manager of Foreign Exchange:
o Manages the Foreign Exchange Management Act, 1999.
o Objective: to facilitate external trade and payment and promote orderly development and maintenance
of foreign exchange market in India.

21 www.visionias.in ©Vision IAS


• Issuer of currency:
o Issues and exchanges or destroys currency and coins not fit for circulation.
o Objective: to give the public adequate quantity of supplies of currency notes and coins and in good
quality.
• Developmental role:
o Performs a wide range of promotional functions to support national objectives.
• Regulator and Supervisor of Payment and Settlement Systems:
o Introduces and upgrades safe and efficient modes of payment systems in the country to meet the
requirements of the public at large.
o Objective: maintain public confidence in payment and settlement system.
• Related Functions:
o Banker to the Government: performs merchant banking function for the central and the state
governments; also acts as their banker.
• Banker to banks:
o maintains banking accounts of all scheduled banks. Hence all the statements are correct.

Q 59.D
• The government has been calculating three indices for CPI :
o CPI(Urban)
o CPI(Rural)
o CPI(combined)
• It is calculated by the National Statistical Office, Ministry of Statistics and Programme
Implementation.
• All items are grouped under 6 categories for CPI(Urban) and 5 categories for CPI(Rural) (Housing
index is zero).
• Maximum weightage is given to Food & Beverages. Hence option (d) is the correct answer.

Q¨V¨LV¢V

Q 60.A
• Inflation is the rise in the general level of prices where a unit of currency effectively buys less than it did
in prior periods. As it pressurised in the form of reducing the purchasing power. Therefore, it is known as
inflationary pressure.
• Increase in interest rate will incentivise the people to spend less and save more because saving
becomes more profitable as interest rates go up. As more and more people choose to save, money is
sucked out of the market and inflation rate moderates. Hence option 1 not is correct.
• Increase in direct tax will result in less disposable income in the hand of people. Therefore people
will have less money to spend money and aggregate demand in the economy will be reduced. It will help
in countering inflationary pressure. Hence option 2 is correct.
• Public expenditure is spending made by the government on collective needs and wants. For example,
pension, provision, infrastructure, etc. Reduction in public spending results in less money in the
market in the hand of people. It will lead to a reduction in the aggregate demand (AD) in the
economy. Therefore, It is helpful in controlling the inflationary pressure. Hence option 3 is correct.

Q 61.C
• Cash Reserve Ratio refers to the fraction of the total Net Demand and Time Liabilities (NDTL) of a
Scheduled Commercial Bank held in India, that it has to maintain a cash deposit with the Reserve Bank of
India (RBI).
22 www.visionias.in ©Vision IAS
• The requirement applies uniformly to all banks in the country irrespective of an individual bank’s
financial situation or size. Hence statement 1 is not correct.
• As per the RBI Act 1934, all Scheduled Commercial Banks (that includes public and private sector banks,
foreign banks, regional rural banks and co-operative banks) are required to maintain a cash balance on
average with the RBI on a fortnightly basis to cater to the CRR requirement. Hence statement 2 is
correct.
• Presently, banks are not paid any interest on behalf of the RBI for parking the required cash. If a bank
fails to meet its required reserve requirements, the RBI is empowered to impose a penalty by charging a
penal interest rate. Hence statement 3 is not correct.

Q 62.D
• A transfer payment is a one-way payment to a person or organization which has given or exchanged no
goods or services for it. This contrasts with a simple "payment," which in economics refers to a transfer of
money in exchange for a product or service.
• Generally, the phrase "transfer payment" is used to describe government payments to individuals through
social programs such as welfare, student grants, and even Social Security. However, government
payments to corporations—including unconditional bailouts and subsidies—are not commonly described
as transfer payments.
• In MGNREGA, a beneficiary receives payment for the manual work which he/she performs. Hence, it
cannot be included in the transfer payment.
• When a farmer receives subsidy under PM Kisan Yojana, he/she performs no work in return. Hence, it is
included in transfer payment.
• When a beneficiary receives a pension under National Old Age Pension Scheme, again he/she is not
performing any work in return. Hence, it is included in transfer payment. Hence option (d) is the
correct answer.

Q 63.C
• Labour Force Participation Rate (LFPR): LFPR is defined as the percentage of persons in the
labour force (i.e. working or seeking or available for work) in the population. The labour force
participation rates is calculated as the labour force divided by the total working-age population.
Hence statement (c) is the correct answer.
o The working age population refers to people aged 15 to 64. This indicator is broken down by age
group and it is measured as a percentage of each age group.
• Worker Population Ratio (WPR): WPR is defined as the percentage of employed persons in the
population. Hence statement (b) is not correct.
• Unemployment Rate (UR): UR is defined as the percentage of persons unemployed among the persons
in the labour force. Hence statement (a) is not correct.

Q 64.B
• Deflation is the economic term used to describe the drop in prices for goods and services. It slows
down economic growth. It generally takes place when there are times of economic uncertainty and
when the demand for goods and services is lower, along with higher levels of unemployment. When
prices fall, the inflation rate drops below 0%. Therefore, a negative growth rate—such as -1%—indicates
deflation, disinflation is demonstrated by a change in the inflation rate from one year to the next. So
disinflation would be measured as a change of 5%.
o In order to deal with deflation, the government adopts the expansionary monetary policy. It lowers
interest rates and increases the money supply within the economy. This, in turn, boosts demand for
goods and services. During the deflation, bonds can perform well. On the other hand, it can have a
negative effect on the stock market. Therefore, investor prefers to invest in bonds than into
equity. Hence option (b) is the correct answer.
• Disinflation is a temporary slowing of the pace of price inflation. It is used to describe instances when the
inflation rate has reduced marginally over the short term.
• Stagflation refers to an economy that has inflation, a slow or stagnant economic growth rate, and a
relatively high unemployment rate.
• Structural inflation is the form of inflation which is due to the operation of the structural weakness
(supply bottleneck, lack of infrastructure, etc.) existing in the economy. Lack of adequate supply
responses or production to increase in demand is the cause of structural inflation. It is mainly phenomena
of developing nations.

23 www.visionias.in ©Vision IAS


Q 65.A
• Market Stabilisation Scheme came into existence in 2004 following an MoU between the Reserve Bank of
India (RBI) and the Government of India (GoI) with the primary aim of aiding the sterilization operations
of the RBI.
• Initially, the MSS was launched to withdraw the excess liquidity in the system that was generated as a
result of the RBI’s purchase of foreign currencies in the foreign exchange market. From 2002 onwards,
there was a huge inflow of foreign capital into India. This led to the appreciation of the rupee. Since
appreciation is not good for exports, the RBI intervened in the foreign exchange market by buying dollars.
To buy dollars, the RBI has to give rupees. In this way, high selling of rupees leads to excess liquidity
(rupee) and thereby creating the potential for inflation. To overcome this situation, the RBI has sold
government bonds on a general basis depending upon the volume of excess liquidity in the system. Here
bonds goto financial institutions and money goes back to the RBI. This withdrawal of excess liquidity is
called sterilization.
• The securities issued under MSS termed as Market Stabilization Scheme (MSS) Securities/Bonds, are
issued by way of auctions conducted by the RBI and are done according to a specified ceiling mutually
agreed upon by the GoI and the RBI. Hence option (a) is not correct.
• Under this scheme, the GoI borrows from the RBI (such borrowing being additional to its normal
borrowing requirements) and issues Treasury-Bills/Dated Securities that are utilized for absorbing excess
liquidity from the market. Hence option (d) is correct.
• Therefore, the MSS constitutes an arrangement aiding in liquidity absorption, in keeping with the overall
monetary policy stance of the RBI, alongside tools like the Liquidity Adjustment Facility (LAF) and Open
Market Operations (OMO). Hence option (b) is correct.
• The amount raised under the MSS does not get credited to the Government Account but is maintained in a
separate cash account with the RBI and are used only for the purpose of redemption/buyback of Treasury-
Bills/Dated Securities issued under the scheme. Hence option (c) is correct.

Q 66.C

The sectoral shares in GVA in 2018-19 in descending order is as follows.


• Financial, Real estate & Professional Services – 21.3
• Trade, Hotel, Transport, Storage, Communication and Services related to broadcasting - 18.3
• Agriculture, forestry and fishing – 16.1
• Manufacturing – 16.4
• Public Administration, Defence and other services – 14.7
• Construction – 8.0
• Electricity, Gas, Water supply and other utility services - 2.8
• Mining and Quarrying – 2.4
It is important to note that in the year 2018-19, the share of manufacturing surpassed that of Agriculture,
forestry and fishing in GVA. Hence option (c) is the correct answer.

24 www.visionias.in ©Vision IAS


Q 67.D
• The government of India has undertaken several expenditure reforms such as simplification of appraisal
and approval processes as well as structural changes in the process of preparing the Budget itself. For
instance, the distinction between plan and non-plan expenditures has been done away with.
• Another key reform has been the development of the Output-Outcome Monitoring Framework
(OOMF). This framework covers major Central Sector (CS) and Centrally Sponsored Schemes
(CSS) with outlays larger than Rs 500 crore, along with clearly defined and measurable output and
outcome indicators as well as targets for the current financial year.
• Historically, budgetary allocations in the government have often not been linked to performance. OOMF
makes a departure from this approach by paving the way for the government’s financial outlays to be
linked to actual delivery of schemes.
• For instance, Ayushman Bharat-Pradhan Mantri Jan Arogya Yojana (PM-JAY), is a centrally-sponsored
health insurance scheme. The measurable outputs for the scheme include number of hospital admissions,
number of identified beneficiaries, amount reimbursed for claims and number of hospitals empanelled.
The targets for each of these respective outputs for the current fiscal year are 50 lakh, 5 crore, 4,000 crore
and 16,500 respectively. The outcome for this scheme is reduction in healthcare expenditures that push
households into poverty. The overall impact of the initiative will be improved access to healthcare for
poor and vulnerable households.
• Monitoring outputs and outcomes enables greater accountability among the executing agencies of various
government schemes. It also promote transparency, predictability and understanding of the government’s
development agenda.
• It supports ministries/departments in tracking achievements against the defined objectives of their
respective schemes.

Q 68.D
• Periodic Labour Force Survey (PLFS) released by the Ministry of Statistics and Programme
Implementation (MoSPI) quarterly basis. Hence option (d) is the correct answer.
• The objective of PLFS is mainly two-fold:
o to estimate the key employment and unemployment indicators (viz. Worker Population Ratio, Labour
Force Participation Rate, Unemployment Rate) in the short time interval of three months for the urban
areas only in the Current Weekly Status (CWS)
o to estimate employment and unemployment indicators in both usual status and Activity Status-
Current Weekly Status (CWS) in both rural and urban areas annually.

Q 69.A
• Gross National Product (GNP) is expressed as:
o GNP = GDP + Factor income earned by the domestic factors of production employed in the rest of the
world – Factor income earned by the factors of production of the rest of the world employed in the
domestic economy. Hence, GNP = GDP + Net factor income from abroad
• A part of the capital gets consumed during the year due to wear and tear. This wear and tear is called
depreciation. Naturally, depreciation does not become part of anybody’s income. If we deduct
depreciation from GNP the measure of aggregate income that we obtain is called Net National Product
(NNP).
• Thus, NNP = GNP – Depreciation
• The expression given above, we get the value of NNP evaluated at market prices. But market price
includes indirect taxes. When indirect taxes are imposed on goods and services, their prices go up.
Indirect taxes accrue to the government. We have to deduct them from NNP evaluated at market prices in
order to calculate that part of NNP which actually accrues to the factors of production. Similarly, there
may be subsidies granted by the government on the prices of some commodities (in India petrol is heavily
taxed by the government, whereas cooking gas is subsidised). So we need to add subsidies to the NNP
evaluated at market prices. The measure that we obtain by doing so is called Net National Product at
factor cost or National Income.
o NNP at factor cost = National Income (NI) = NNP at market prices –(Indirect taxes – Subsidies) =
NNP at market prices – Net indirect taxes (Net indirect taxes = Indirect taxes – Subsidies).
• Hence option (a) is the correct answer.

25 www.visionias.in ©Vision IAS


Q 70.C
• Revenue expenditures are the ongoing operating expenses, which are short-term expenses used to run the
daily business operations. It neither creates assets nor reduces a liability. e.g., salaries of employees,
interest payment on past debt, subsidies, pension, etc.
• Capital expenditures are typically one-time large purchases of fixed assets that will be used for revenue
generation over a longer period. It creates an asset (e.g., hospital building) or reduces a liability (e.g.,
repayment of loan).
• Government Budget separately classifies the above expenditure and under several subheads. The order of
the revenue expenditures and actual expenditure incurred in 2018-19 are as follows (in rupees)
• Interest Payment and Servicing of Debt - 595552.17 crores
• Transport- 214548.73 crores
• Defence Services - 203882.36 crores
• Agriculture and Allied Activities 176677.96 crores
• Hence option (c) is the correct answer.

Q 71.C
• The Central Statistics Office (CSO)[ Now, National Statistical Office] of the Government of India has
been reporting the GDP at factor cost and at market prices. In its revision, in January 2015 the CSO
replaced GDP at factor cost with the GVA at basic prices, and the GDP at market prices, which is now
called only GDP.
• Gross value added is the output of the country less the intermediate consumption, which is the difference
between gross output and net output.
• The distinction between factor cost, basic prices and market prices is based on the distinction between net
production taxes (production taxes less production subsidies) and net product taxes (product taxes less
product subsidies).
• Factor cost includes only the payment to factors of production, it does not include any tax. In order to
arrive at the market prices, we have to add to the factor cost the total indirect taxes less total subsidies.
The basic prices lie in between: they include the production taxes (less production subsidies) but not
product taxes (less product subsidies). Therefore in order to arrive at market prices we have to add product
taxes (less product subsidies) to the basic prices.
• So, GVA at factor costs + Production taxes - Production subsidies = GVA at basic prices
• GVA at basic prices + Product taxes - product subsidies = GDP at market prices.
• Hence, option (c) is the correct answer.

Q 72.A
• When interest rates go up, people become less interested in holding money since holding money amounts
to holding less of interest-earning deposits, and thus less interest received. Therefore, at higher interest
rates, money demanded comes down. Hence statement 1 is correct.
• The policy rate is the key lending rate of the central bank in a country. It is a monetary policy instrument
under the control of the Central Bank. In India, it is decided by the Reserve Bank of India. Hence
statement 2 is not correct.

Q 73.A
• Cash Deposit Ratio: The currency deposit ratio (cdr) is the ratio of money held by the public in
currency to that they hold in bank deposits. If a person gets Re 1 she will put Rs 1/(1 + cdr) in her bank
account and keep Rs cdr/(1 + cdr) in cash.
• The Reserve Deposit Ratio: Banks hold a part of the money people keep in their bank deposits as reserve
money and loan out the rest to various investment projects. Reserve money consists of two things – vault
cash in banks and deposits of commercial banks with RBI. Banks use this reserve to meet the demand for
cash by account holders. Reserve deposit ratio (rdr) is the proportion of the total deposits commercial
banks keep as reserves.

Q 74.C
• Recent Context: Iran Plans To Enrich Uranium To Up To 20% Purity In Violation Of 2015 Deal.
• The Joint Comprehensive Plan of Action (JCPOA) known commonly as the Iran nuclear deal or
Iran deal, is an agreement on the Iranian nuclear program reached in Vienna on 14 July 2015, between
Iran and the P5+1 (the five permanent members of the United Nations Security Council—China,
France, Russia, United Kingdom, United States—plus Germany) together with the European Union.
• In 2018, the USA has withdrawn from this accord.
26 www.visionias.in ©Vision IAS
• Under the accord, some of the limitations that were placed on Iran are-
o Iran agreed to limit its sensitive nuclear activities and allow in international inspectors in return for
the lifting of crippling economic sanctions. Under the JCPOA, the centrifuges was limited to
installing no more than 5,060 of the oldest and least efficient centrifuges at Natanz until 2026 - 10
years after the deal's "implementation day" in January 2016.
o Iran's uranium stockpile was reduced by 98% to 300kg (660lbs), a figure that must not be exceeded
until 2031. It must also keep the stockpile's level of enrichment at 3.67%.
o In addition, research and development must take place only at Natanz and be limited until 2024.
o Periodic review by International Atomic Energy Agency.
• Hence option (c) is the correct answer.

Q 75.C
• The Central Government has released the Ninth Edition of the Status Paper on the Government Debt
which provides a detailed analysis of the Overall Debt Position of the Government of India.
• The Central Government has been bringing-out a Status Paper on Government Debt since 2010-11.
• Central Government’s Debt as a percentage of GDP has dropped marginally by 0.1% from 45.8% in
fiscal 2017-18 to 45.7% in fiscal 2018-19.
• Gross Fiscal Deficit (GFD) as a percentage of GDP has been on a declining trend since 2012-13.
o The share of short-term debt is within safe limits and has stabilised after some rising from
2005 to 2012. Hence, statement 2 is correct.
o The Government has adopted a conscious strategy of elongating maturity to reduce roll-over risk.
o 69.9 per cent of total securities issued during 2018-19 were in the maturity bucket of 10 years
and above.
o Most of the Government debt is at fixed interest rates, with floating internal debt constituting only
0.9 per cent of GDP in 2019, which minimises the impact of interest rate volatility on the
budget. Hence, statement 1 is correct.
o The low share of external debt implies that currency risk and the susceptibility of debt portfolio
to volatile international capital markets is not substantial.
o The ratio of interest payments to revenue receipts (IP-RR) of the Central Government was 37.5per
cent in 2018-19 as compared to 35.6 per cent in 2012-13
• Government’s finances were largely protected from currency risks as external debt stood at 2.7% of GDP
or Rs 5.12 lakh crore in FY19.

Q 76.C
• Recent Context: The on-going negotiation to curb harmful fisheries subsidies at the World Trade
Organisation (WTO) is facing a deadlock due to deeply fixed positions of members over special and
differential treatment (S&DT) concessions for developing countries. India, South Africa and some
others are insisting on carve-outs for poorer nations to protect the subsidies of small fishers.
• The WTO Agreements contain special provisions that give developing countries special rights and which
give developed countries the possibility to treat developing countries more favourably than other WTO
Members. These special provisions include, for example, longer time periods for implementing
Agreements and commitments or measures to increase trading opportunities for developing countries.
o These provisions are referred to as “special and differential treatment” (S&D) provisions. Hence
option (c) is the correct answer.
• The special provisions include:
o longer time periods for implementing Agreements and commitments,
o measures to increase trading opportunities for developing countries,
o provisions requiring all WTO members to safeguard the trade interests of developing countries,
o support to help developing countries build the capacity to carry out WTO work, handle disputes, and
implement technical standards, and
o provisions related to least-developed country (LDC) Members.
• In the Doha Declaration, member governments agreed that all special and differential treatment
provisions are an integral part of the WTO agreements. The Bali Ministerial Conference in December
2013 established a mechanism to review and analyze the implementation of special and differential
treatment provisions.

27 www.visionias.in ©Vision IAS


Q 77.C
• Neom is a planned cross-border city in the Tabuk Province of northwestern Saudi Arabia. It is
planned to incorporate smart city technologies and also function as a tourist destination. The site is near
the Red Sea, and Strait of Tiran and the borders of Egypt, Israel and Jordan.
• NEOM means "New Future". The acronym NEOM was first used publicly with the meaning "New
Future" on 24 October 2017, when Saudi Crown Prince Mohammed bin Salman announced the launch of
a project involving the construction of a futuristic $500 billion city that will be built to run entirely on
alternative energy.
• Hence option (c) is the correct answer.

Q 78.C
• Cost-Push Inflation means when the aggregate supply of goods and services decreases because of an
increase in production costs Cost-push inflation means prices have been "pushed up" by increases in the
costs of any of the four factors of production—labour, capital, land, or entrepreneurship when companies
are already running at full production capacity. Therefore, an increase in the cost of wages and raw
materials will result in cosh-push inflation. Hence statements 1 and 3 are correct.
• The unexpected causes of cost-push inflation are often natural disasters, which can include cyclones,
floods, earthquakes and fires etc. If a large disaster causes unexpected damage to a production facility
and results in a shutdown or partial disruption of the production chain, higher production costs are likely
to follow. A company might have no choice but to increase prices to help recoup some of the losses from
a disaster. .
• Expansionary monetary policy is a form of macroeconomic monetary policy, under which central
bank seeks increases the money supply to stimulate the economy. It results in an increase in money
supply and aggregate demand in the economy consequently. Therefore, it leads to demand-pull
inflation. Hence statement 2 is not correct.

Q 79.A
• The interest rate paid by the banks to depositors is lower than the rate charged from the borrowers. This
difference between these two types of interest rates, called the ‘spread’ is the profit appropriated by the
bank. Hence option (a) is correct.
• Also called the net interest spread, the bank spread is a percentage that tells someone how much money
the bank earns versus how much it gives out. A bank earns money from interest it receives on loans and
other assets, and it pays out money to customers who make deposits into interest-bearing accounts.
• The ratio of money it receives to money it pays out is called the bank spread.The bank spread can indicate
a bank’s profit margin. A high spread equates to a higher profit margin, since the difference between
interest earned and interest paid out is high.
• However, bank spread measures the average difference between lending and borrowing interest rates, not
the amount of banking activity itself, which means that bank spread doesn’t necessarily indicate a
financial institution’s profitability.
• Consider a bank that lends money to customers at an average rate of 8 percent. At the same time, the
interest rate the bank pays on funds that customers deposit into their personal accounts is 5 percent.
• The net interest spread of that financial institution would be 8 percent minus 5 percent, resulting in a bank
spread of 3 percent.

Q 80.A
• HSN stands for Harmonized System of Nomenclature
• It was developed by the World Customs Organization (WCO) with the vision of classifying goods all
over the World in a systematic manner.
• This brings in a uniform classification of goods and facilitates international trade.
• The HSN system is used by more than 200 countries and economies for reasons such as Uniform
classification, Base for their Customs tariffs, Collection of international trade statistics.
• Over 98% of the merchandise in international trade is classified in terms of the HSN.
• HSN code was adopted by India to make GST standardized and worldwide accepted
• HSN is a 4 to 8-digit code for identifying the applicable rate of GST on different products as per CGST
rules of the government of India.
• HSN Code is to be mentioned in GST invoices.
• Service Accounting Code (SAC)
28 www.visionias.in ©Vision IAS
• It is a unique code that is identical to the HSN codes.
• SACs are issued by Central Board of Indirect Tax & Customs for the classification of services. Hence,
option (a) is not correct.

Q 81.D
• Hard currency- It is the international currency in which the highest faith is shown and is needed by every
economy. The strongest currency of the world is one which has a high level of liquidity. Basically, the
economy with the highest as well as highly diversified exports that are compulsive imports for other
countries (as of high-level technology, defence products, life-saving medicines and petroleum products)
will also create high demand for its currency in the world and become the hard currency. It is always
scarce. Some of the best hard currencies of the world today are the US Dollar, the Euro(€), Japanese
Yen(¥), and the UK Sterling Pound (£). Meanwhile, by late 2015, the IMF allowed the SDR to be
denominated in the Chinese ‘Yaan’–paving the way for a new hard currency to be implemented in 2016.
• Heated currency- A term used in the forex market to denote the domestic currency which is under
enough pressure (heat) of depreciation due to a hard currency’s high tendency of exiting the economy
(since it has become hot). It is also known as currency under heat or under hammering.
• Dear currency- This term was popularised by economists in the early 1930s to show the opposite of
cheap currency. When a government issues bonds, the money which flows from the public to the
government or the money in the economy, in general, is called dear currency, also called dear money. In
the banking industry, it means a period of comparatively higher/costlier interest rates regime.

Q 82.D
• Recent Context: India Infrastructure Finance Company Ltd (IIFCL) a Special Purpose Vehicle for
Financing Viable Infrastructure Projects was given the tag of Development Financial institution (DFI).
• A Development Financial Institution (DFI) is defined as “an institution endorsed or supported by
the Government of India primarily to provide development/Project finance to one or more sectors
or sub-sectors of the economy. the institution differentiates itself by a thoughtful balance between
commercial norms of operation, as adopted by any financial institution like a commercial bank and
developmental responsibilities.
• It emphasizes the long term financing of a project rather than collateral-based financing apart from the
provision of long-term loans, equity capital, guarantees and underwriting functions, a development
institution normally is also expected to upgrade the managerial and the other operational requirements of
the assisted projects.
• India’s first DFI was operationalized in 1948 and it set up State Financial Corporations (SFCs) at the state
level after passing of the SFCs Act, 1951, succeeded by the development of the Industrial Finance
Corporation of India (IFCI).
• DFIs can be classified into four categories of institutions as per their functions:
o National Development Banks e.g. IDBI, SIDBI, ICICI, IFCI, IRBI, IDFC
o Sector-specific financial institutions e.g. TFCI, EXIM Bank, NABARD, HDFC, NHB
o Investment Institutions e.g. LIC, GIC, and UTI
o State-level Institutions e.g. State Finance corporations and SIDCs
• Specialized development financial institutions (DFIs), such as Industrial Finance Corporation of India
(IFCI), Industrial Development Bank of India (IDBI), National Bank for Agriculture and Rural
Development (NABARD), National Housing Board (NHB) and Small Industry Development Bank of
India (SIDBI), with majority ownership of the RBI were launched to meet the long-term financing needs
of industry and agriculture in India for driving growth in our economy post-independence.
• But RBI does not engage in direct financing. There is no specific use of the term 'DFI' in either the
RBI Act, 1934 or the Companies Act, 1956 or various statutes establishing DFIs.
• Hence option (d) is the correct answer.
Q 83.A
• A Non-Banking Financial Company (NBFC) is a company registered under the Companies Act, 1956
engaged in the business of loans and advances, acquisition of shares/stocks/bonds/debentures/securities
issued by Government or local authority or other marketable securities of a like nature, leasing, hire-
purchase, insurance business, chit business but does not include any institution whose principal business is
that of agriculture activity, industrial activity, purchase or sale of any goods (other than securities) or
providing any services and sale/purchase/construction of the immovable property.
29 www.visionias.in ©Vision IAS
• These are either registered by SEBI or RBI. RBI regulates the companies which deal in lending, hire
purchase and acquisition of shares/stocks, etc.
• To obviate dual regulation, certain categories of NBFCs which are regulated by other regulators are
exempted from the requirement of registration with RBI viz. Venture Capital Fund/Merchant Banking
companies/Stockbroking companies registered with SEBI, Insurance Company holding a valid Certificate
of Registration issued by IRDA, Nidhi companies as notified under Section 620A of the Companies Act,
1956, Chit companies as defined in clause (b) of Section 2 of the Chit Funds Act, 1982, Housing Finance
Companies regulated by National Housing Bank, Stock Exchange or a Mutual Benefit company. Hence
statement 1 is correct.
• NBFCs lend and make investments and hence their activities are akin to that of banks; however there are a
few differences as given below: NBFC cannot accept demand deposits; NBFCs do not form part of the
payment and settlement system and cannot issue cheques drawn on itself; deposit insurance facility of
Deposit Insurance and Credit Guarantee Corporation is not available to depositors of NBFCs, unlike in
case of banks. Hence statement 2 is not correct.

Q 84.A
• Hidden unemployment is also known as disguised unemployment.
• It can refer to any segment of the population not employed at full capacity, but it is often not counted in
official unemployment statistics within the national economy. This can include those working well below
their capabilities, those whose positions provide little overall value in terms of productivity, or any group
that is not currently looking for work but is able to perform work of value.
• Structural unemployment occurs when certain industries decline because of long term changes in
market conditions. For example, over the last 20 years, UK motor vehicle production has declined while
car production in the Far East has increased, creating structurally unemployed car workers. Globalisation
is an increasingly significant cause of structural unemployment in many countries.
• Cyclical unemployment exists when individuals lose their jobs as a result of a downturn in aggregate
demand (AD). If the decline in aggregate demand is persistent, and the unemployment long-term, it is
called either demand deficient, general, or Keynesian unemployment. Cyclical unemployment is the
component of overall unemployment that results directly from cycles of economic upturn and downturn.
Unemployment typically rises during recessions and declines during economic expansions. For example,
unemployment levels of 3 million were reached in the UK in the last two recessions, between 1980 and
1982, and between 1990 and 1992. In the most recent recession of 2008-2010, unemployment levels rose
to 2.4m in the last quarter of 2009, and are likely to peak at over 2.5m during 2010.
• Voluntary unemployment is defined as a situation when workers choose not to work at the current
equilibrium wage rate. For one reason or another, workers may elect not to participate in the labour
market. There are several reasons for the existence of voluntary unemployment including excessively
generous welfare benefits and high rates of income tax. Voluntary unemployment is likely to occur when
the equilibrium wage rate is below the wage necessary to encourage individuals to supply their labour.
• Hence option (a) is the correct answer.

Q 85.B
• Open market operations is the sale and purchase of government securities and treasury bills by RBI or
the central bank of the country. Hence option 1 is correct.
• The objective of OMO is to regulate the money supply in the economy. Hence it is used to control
liquidity and inflation.
• Bank rate is the rate at which the central bank lends money to the commercial banks by buying their
eligible rated securities - bills of exchange or commercial paper.
• Bank Rate once used to be the policy rate (the key interest rate based on which all other short term interest
rates move) in India. Hence option 2 is correct.
• A rise in bank rate raises the deposit as well as lending rates in the economy while a lowering of the bank
rate reduces these rates.
• It is the central banker’s tool for controlling liquidity and inflation. Hence it is used to control liquidity
and inflation.
• The repo rate is the rate at which the RBI lends money to the banking system (or banks) for short
durations. The reverse repo rate is the rate at which banks can park their money with the RBI. It is often
used to control liquidity and inflation. Hence option 3 is correct.

30 www.visionias.in ©Vision IAS


• Increasing the tax base can improve the fiscal conditions of the Government. It is a tool of the Central
Government. Hence option 4 is not correct.

Q 86.C
• We may be tempted to treat higher level of GDP of a country as an index of greater well-being of the
people of that country (to account for price changes, we may take the value of real GDP instead of
nominal GDP). But this is not the case. A higher level of GDP cannot be regarded as an index of greater
wellbeing of the people of a country.
• Distribution of GDP may not be uniform - If the GDP of the country is rising, the welfare may not rise as
a consequence. This is because the rise in GDP may be concentrated in the hands of very few individuals
or firms. For the rest, the income may in fact have fallen. In such a case the welfare of the entire country
cannot be said to have increased. Hence statement 1 is correct.
• Growth of GDP results in rise of Direct taxes – Increasing or decreasing of direct taxes is a fiscal policy
matter which has no direct connection with the GDP. So this statement does not explain as to why GDP
cannot be regarded as an index of the wellbeing of people. Hence statement 2 is not correct.
• Many activities in an economy are not evaluated in monetary terms. Many activities in an economy are
not evaluated in monetary terms. For example, the domestic services women perform at home are not paid
for. The exchanges which take place in the informal sector without the help of money are called barter
exchanges. So, GDP calculated in the standard manner may not give us a clear indication of the
productive activity and well-being of a country. Hence statement 3 is correct.
• GDP inherently does not take into account negative externalities. Externalities refer to the benefits (or
harms) a firm or an individual causes to another for which they are not paid (or penalised). Externalities
do not have any market in which they can be bought and sold. In carrying out the production the refinery
may also be polluting the nearby river. This may cause harm to the people who use the water of the river.
Hence their well being will fall. Pollution may also kill fish or other organisms of theriver on which fish
survive. As a result, the fishermen of the river may be losing their livelihood. Such harmful effects that
the refinery is inflicting on others, for which it will not bear any cost, are called externalities. Hence
statement 4 is correct.
Q 87.C
• Recent Context: One more wetland in India has been added to the list of recognised sites of international
importance under the treaty of Ramsar Convention, taking the number of such wetlands in the country to
42, the highest in South Asia.
o The latest site to be added to the list is a high-altitude wetland complex of two connected lakes,
Startsapuk Tso and Tso Kar, in Ladakh. It is a high-altitude wetland complex in Changthang
region of Ladakh. The complex is a notable example of two connected lakes, the freshwater
Startsapuk Tso and the hypersaline Tso Kar.
o Last month, two wetlands -- the Lonar lake in Maharashtra and Sur Sarovar, also known as Keetham
lake, in Agra -- were added to the list of Ramsar sites.
o Before that, Kabartal in Bihar's Begusarai district was recognised as a wetland of international
importance under the Ramsar Convention. It was the first such site in the state to figure in the list.
o The Asan Conservation Reserve in Dehradun, the first wetland from Uttarakhand to be recognised by
Ramsar, was added to the list in October this year.
• The Tso-Kar Lake also known as 'White Lake' is one of the three high altitude salt water lakes in
Ladakh. It is known as 'White Lake' because the white salt of the water deposits all over the lake shores.
Tso-kar Lake is the smallest lake among the three important lakes in Ladakh. It is situated in Rupshu
Valley, about 250 kms southeast from its principle town.
• Tsomoriri lake is located to the southeast of Leh in eastern Ladakh and it is saline water lake. The
lake is at an altitude of 4,522m (14,836ft). It is the largest of the high altitude lakes entirely within India
and entirely within Ladakh in Trans-Himalayan biogeographic region.
• Pangong Tso lake is also a brackish water lake located on boundary of India and China, and comes in the
Ladakh region.
• However, Chandra taal lake is situated in the Lahul and Spiti of Himachal Pradesh.
• Hence option (c) is the correct answer.

Q 88.C
• Recent Context: The Vadodara Municipal Corporation (VMC) is expected to launch municipal bonds in
January 2021, and will become the third Urban Local Body (ULB) in Gujarat to use this method to raise
money to fund development work sanctioned under the Atal Mission for Rejuvenation and Urban
Transformation (AMRUT). It has sought approval for the same from the state government.
31 www.visionias.in ©Vision IAS
• The urban local government and agencies issue these bonds. Municipal bonds exist in India since the
year 1997. Bangalore Municipal Corporation is the first urban local body to issue municipal bonds in
India. Ahmedabad followed Bangalore in the succeeding years.
• The Securities and Exchange Board of India (SEBI) circulated detailed guidelines in 2015 for the
urban local bodies to raise funds by issuing municipal bonds.
• A municipality should meet the following eligibility criteria to issue municipal bonds in India (hence
statement 1 is not correct):
o The municipality must not have a negative net worth in each of the three previous years.
o The municipality must have no default in the repayment of debt securities and loans availed from the
banks or non-banking financial companies in the last year.
o The municipality, promoter and directors must not be enlisted in the wilful defaulters published by the
Reserve Bank of India (RBI). The municipality should have no record of default in the payment of
interest and repayment of principal with respect to debt instruments.
• Municipal bonds are also referred to as ‘muni bonds’. They are issued when a government body
wants to raise funds for projects such as infra-related, roads, airports, railway stations, schools, and
so on. Hence statement 2 is not correct.

Q 89.A
• The Phillips curve is an economic concept developed by A. W. Phillips stating that inflation and
unemployment have a stable and inverse relationship. Hence option (a) is the correct answer.
o The theory claims that economic growth
comes inflation, which in turn should lead to
more jobs and less unemployment.
o The concept behind the Phillips curve states
the change in unemployment within an
economy has a predictable effect on price
inflation.
o The inverse relationship between
unemployment and inflation is depicted as a
downward sloping, concave curve, with
inflation on the Y-axis and unemployment on
the X-axis.
• The Phillips Curve and Stagflation: Stagflation
refers to stagnant economic growth with high
unemployment and high price inflation. This
scenario directly contradicts the concept of Philips curve. Hence option (c) is not correct.
• The Laffer Curve is a theory developed by supply-side economist Arthur Laffer to show the relationship
between tax rates and the amount of tax revenue collected by governments.
o The curve is used to illustrate Laffer's argument that sometimes cutting tax rates can increase total tax
revenue.
o If taxes are too high along the Laffer Curve, then they will discourage the taxed activities, such as
work and investment, enough to actually reduce total tax revenue. In this case, cutting tax rates will
both stimulate economic incentives and increase tax revenue. Hence option (b) is not correct.
Q 90.B
• Recent Context: Currently, NRIs who hold an Indian passport, can vote physically in his/her home town
after registering as an Overseas Voter. ECI has recommended to extend ETPBS facility to NRIs.
• Electronically Transmitted Postal Ballot System (ETPBS) is developed by Election Commission of
India with the help of Centre for Development of Advanced Computing (C-DAC), for the use of the
Service Voters. Hence statement 1 is not correct.
• It is a fully secured system, having two layers of security. Secrecy is maintained through the use of
OTP and PIN and no duplication of casted Electronically Transmitted Postal Ballot (ETPB) is
possible due to the unique QR Code.
• ETPBS is a fully secured system, having two security layers. The secrecy of voting is maintained
through the use of OTP and PIN and no duplication of casted ETPB is possible due to the unique QR
Code in the portal https://www. etpbs.in. Through this system the service voters cast their vote on an
electronically received postal ballot, from anywhere outside their constituency, thus reducing the
chances of losing the voting opportunity. The purpose of the online system was to create convenient and
easy-to-use online system for Defense Personnel to become Service Voters. Hence statement 2 is
correct.
32 www.visionias.in ©Vision IAS
• Service Voter turnout in 2014 was only 4%. Compared to 13,27,627 number of registered Service Electors
of last General Election in 2014, a record highest number of 18,02,646 were enrolled as Service Electors
in 2019 in the world’s largest democratic election process. For the first time enabled through the dedicated
portal (https://www.servicevoter.eci.nic.in) online registration, the service voters were sent postal ballots
electronically one way to save processing time, resources and avoid human errors.
• Persons working in Central Forces under Arms Act and Government officials deployed in
Embassies outside the country are classified as Service Voters and are provisioned for online
enrolment.

Q 91.A
• The Gross Domestic Product (GDP) deflator is a measure of general price inflation. It is calculated by
dividing nominal GDP by real GDP and then multiplying by 100. Nominal GDP is the market value of
goods and services produced in an economy, unadjusted for inflation (It is the GDP measured at current
prices). Real GDP is nominal GDP, adjusted for inflation to reflect changes in real output.
• There are other measures of inflation too like Consumer Price Index (CPI) and Wholesale Price Index (or
WPI); however GDP deflator is a much broader and comprehensive measure. Since Gross Domestic
Product is an aggregate measure of production, being the sum of all final uses of goods and services (less
imports), GDP deflator reflects the prices of all domestically produced goods and services in the economy
whereas, other measures like CPI and WPI are based on a limited basket of goods and services, thereby
not representing the entire economy. Hence, statement 1 is correct.
• Another important distinction is that the basket of WPI (at present) has no representation of the services
sector. The GDP deflator also includes the prices of investment goods, government services and exports,
and excludes the price of imports. Changes in consumption patterns or the introduction of new goods and
services or structural transformation are automatically reflected in the deflator which is not the case with
other inflation measures.
• However, WPI and CPI are available on a monthly basis whereas deflator comes with a lag (yearly or
quarterly, after quarterly GDP data is released). Hence, the monthly change in inflation cannot be
tracked using the GDP deflator, limiting its usefulness. Hence, statement 2 is not correct.
Q 92.A
• Fiscal policy refers to the use of government spending and tax policies to influence economic conditions,
especially macroeconomic conditions, including aggregate demand for goods and services, employment,
inflation, and economic growth.
• Fiscal policy helps to stabilize the economy during business cycles. The two important phases of business
cycles are boom and recession. A recession should not be allowed to grow into a deep recession.
Similarly, a boom should not explode bigger. Fiscal policy strategies of taxation and expenditure can go in
two ways in response to the business cycle: Countercyclical and procyclical.
• Counter-cyclical fiscal policy: It refers to strategy by the government to counter boom or recession
through fiscal measures. It works against the ongoing boom or recession trend; thus, trying to stabilize the
economy. Understandably, countercyclical fiscal policy works in two different direction during these two
phases.
o Recession: Recession is a business cycle situation where there is slowing demand and falling growth
in the economy. Here, the Government’s responsibility is to generate demand by fine-tuning taxation
and expenditure policies. Reducing taxes and increasing expenditure will help to create demand and
producing upswing in the economy.
o Boom: In the case of boom, economic activities will be on upswing. Amplifying the boom is
disastrous as it may create inflation and debt crisis and the government’s responsibility here is to
bring down the pace of economic activities. Increasing taxes and reducing public expenditure will
make boom mild. Thus, slowing down demand should be the nature of countercyclical fiscal policy
during boom. Hence only options 1 and 2 are correct.
• Procyclical fiscal policy: Itis the opposite of countercyclical. Here, fiscal policy goes in line with the
current mood of the business cycle; amplifying them. For example, during the time of boom, government
makes high expenditure and doesn’t hike taxes. Thus, boom grows further. Such a policy is dangerous and
tends to bring instability in the economy.
o Boom: total government spending as a percentage of GDP goes up and tax rates go down, increasing
government deficit.
o Recession: total government spending as a percentage of GDP goes down and tax rates go up,
decreasing government deficit.
33 www.visionias.in ©Vision IAS
Q 93.A
• On the basis of time and willingness criteria open unemployment and underemployment have been
estimated using the following three approaches:
• Usual Status Approach: This approach records only those persons as unemployed who had no gainful
work for a major time during the 365 days (one year) preceding the date of the survey and are seeking or
are available for work. Thus, the estimates of unemployment obtained on the basis of the usual status
approach are expected to capture long-term open unemployment. Hence option (a) is the correct
answer.
• Weekly Status Approach: In this approach, current activity status relating to the week preceding the date
of the survey is recorded and those persons are classified as unemployed who did not have gainful work
even for an hour on any day in the preceding week and were seeking or were available for work.
• Daily Status Approach: The weekly status approach records a person employed even if he works only
for an hour on any day of the whole week. It is thus clear that the weekly status approach would tend to
underestimate unemployment in the economy because it does not appear to be proper to treat all those
who have been unemployed for the whole week except an hour as employed. Indeed, the demand for
labour in farming and non-fanning households often fluctuates over a small period within a
week. Hence the need for the use of daily status approach to measure the magnitude of unemployment and
underemployment in India. In the daily status approach current activity status of a person with regard to
whether employed or unemployed or outside labour force is recorded for each day in the reference week.
Further, for estimating employment and unemployment, half-day has been adopted as a unit of
measurement.

Q 94.C
• Capital is a vital ingredient for economic growth since most nations cannot meet their total capital
requirements from internal resources alone, they turn to foreign investors. Foreign direct investment (FDI)
and foreign portfolio investment (FPI) are two of the most common routes for investors to invest in an
overseas economy. FPI means investing in financial assets, such as stocks and bonds of entities
located in another country.
• There are certain factors which favour foreign investments such as:
o Political factors: political stability, favourable business policy.
o Economic factors: the strength of the economy, GDP growth trends, infrastructure, inflation, lower
currency risk.
• As inflation is usually coupled with the economic growth and it results in higher return in the form
of the equity for the investor. Inflation also affects the exchange rate; it leads weaker of domestic
currency with respect to investor’s hot currency. Therefore, it results in better returns for foreign
investors in the form of exchange rate who had invested in the host nation’s financial assets.
• Hence both statements 1 and 2 are correct.
Q 95.D
• In one of the biggest consolidation exercise in the banking space, the government in August 2019
announced the merger of 10 public sector lenders into four bigger and stronger banks. With this, the
number of public sector banks in India will come down to 12 from 27in 2017. The government
consolidated 10 existing banks, including Punjab National Bank, the country’s second-largest state lender,
into four entities in a bid to get credit flowing into the economy.
• It falls under the ambit of the Government of India. It is done after the approval by the Union
cabinet. Hence statement 1 is not correct.
• In a merger, there is an anchor bank and an amalgamating bank or banks, where the latter gets merged
with the former. Here a new entity is not formed. Hence statement 2 is not correct.

Q 96.A
• To encourage female participation in the economy, various programmes/ legislative reforms are in place.
A number of protective provisions have been incorporated in various labour laws for creating a congenial
work environment for women workers such as child care centres, time-off for feeding children,
enhancement in paid maternity leave from 12 weeks to 26 weeks, provisions for mandatory crèche facility
in the establishments having 50 or more employees, permitting women workers in the night shifts with
adequate safety measures, etc.
34 www.visionias.in ©Vision IAS
• One-Stop Centre (OSC): This scheme facilitates access to an integrated range of services including
police, medical, legal, psychological support and temporary shelter to women affected by violence. One-
Stop Centres are being set up in all districts of the country.
• Prime Minister’s Employment Generation Programme (PMEGP): Under the scheme, women
entrepreneurs are provided 25 per cent and 35 per cent subsidies for the project set up in urban and rural
areas respectively. For women beneficiaries, their own contribution is only 5 per cent of the project cost
while for the general category it is 10 per cent. All the entrepreneurs including women are eligible for 2
weeks Entrepreneurship Development Programme (EDP) after the sanction of their projects from banks
for financial assistance to set up their unit and also imparting training under EDP
• Deendayal Antyodaya Yojana National Rural Livelihoods Mission (DAY-NRLM) - seeks to reach
out to 8-9 crore rural poor households and organize one-woman member from each household into
affinity-based women SHGs and federations at the village and at higher levels. Support is provided to the
women for employment and self-employment venture under the scheme to enhance their farm livelihoods
and non-farm livelihood through various components of schemes such as organic farming, Krishi Sakhi,
Pashu Sakhi, dairy value chain interventions, Aajeevika Grameen Express Yojana, Start-up Village
Entrepreneurship Programme etc. DAY-NRLM provides Revolving Funds (RF) at the rate of ` 10,000-
15,000 per SHG and Community Investment Support Fund(CISF) to the extent of ` 2.5 lakh per SHG to
support their income-generating and livelihood activities.
• Rashtriya Mahila Kosh (RMK) is an apex micro-finance organization that provides micro-credit at
concessional terms to poor women for various livelihood and income-generating activities. Among other
things, RMK also promotes and supports the expansion of entrepreneurship skills among women.
• Hence option (a) is the correct answer.

Q 97.B
• FPI stands for those investors who hold a short term view of the company, in contrast to Foreign Direct
Investors (FDI).
• FPIs generally participate through the stock markets and get in and out of a particular stock at much faster
frequencies.
• Short term view is associated often with a lower stake in companies. Hence, globally FPIs are defined as
those who hold less than 10% in a company. Hence statement 1 is correct.
• FPIs are not allowed to invest in unlisted shares. However, all existing investments made by the FIIs/Sub-
accounts/QFIs are grandfathered.
• In respect of those securities, where FPIs are not allowed to invest no fresh purchase shall be allowed as
FPI. They can only sell their existing investments in such securities. Hence statement 2 is not correct.
• Capital account can be regarded as one of the primary components of the balance of payments of a nation.
• It gives a summary of the capital expenditure and income for a country. It includes the following: Foreign
Direct Investment (FDI), Foreign Portfolio Investment (FPI), Other investments, Reserve Account. Hence
statement 3 is correct.
Q 98.C
• Recently a Tawang-based social worker had written to the Khadi and Village Industries
Commission (KVIC) bringing to its notice the potential of a rare and languishing paper-making
craft of his state. ‘Mon shugu’, or the paper of the Monpa people is a traditional form of paper
making technque.
• In the forests of Mukto, a village perched at an altitude of 10,800 feet in Tawang district in Arunachal
Pradesh, grows the shugu sheng shrub (Daphne papyracea), the bark of which has been traditionally
processed into ‘mon shugu’ by the Monpa tribe. The bark from the shrub has to be extricated, dried,
boiled with a solution of ash, made into pulp and then cut into sheets of paper. Hence option (c) is the
correct answer.
• The paper has made its way to the many Buddhist monasteries not just locally, but in Tibet, Bhutan, China
and Japan too, where it serves as a medium for religious scriptures, manuscripts, prayer flags, and
sometimes as part of flag poles and prayer wheels. The paper has a huge tensile strength, is durable, and
made without a single chemical additive.
• Many communities here have local laws such as not allowing their forest produce to be taken outside the
villages. So it may be difficult to get an unlimited supply of shugu sheng, which grows only in certain areas

35 www.visionias.in ©Vision IAS


Q 99.A
• An AI & Robotics Technologies Park (ARTPARK) set up in Bengaluru will promote technology
innovations in AI (Artificial Intelligence) & Robotics leading to societal impact by executing ambitious
mission mode R&D projects in healthcare, education, mobility, infrastructure, agriculture, retail and
cyber-security focusing on problems unique to India.
• ARTPARK, is a unique not-for-profit foundation established by Indian Institute of Science (IISc),
Bengaluru with support from AI Foundry in a public-private model. With seed funding of Rs. 170 Cr
($22mn) from Department of Science & Technology (DST), Govt. of India, under the National Mission
on Inter-disciplinary Cyber-Physical Systems (NM-ICPS), it will bring about collaborative consortium of
partners from industry, academia and government bodies. This will lead to cutting edge innovations in
terms of new technologies, standards, products, services and intellectual properties.
• ARTPARK will develop AI & Robotics facilities to support technology innovations as well as capacity
building through advanced skills training of students and professionals in these areas. It will develop
DataSetu - that will enable confidentiality and privacy-preserving framework to share data and run
analytics spurring the data-sharing ecosystem and create a data marketplace, boosting AI applications and
solutions. One such service will be BhashaSetu - that will enable real-time Indic language translation,
both of speech to speech and speech to text.
• Hence option (a) is the correct answer.

Q 100.C
• The Ministry of Earth Sciences (MoES) is developing an ‘Early Health Warning System’ which is
expected to forecast the possibility of disease outbreaks in the country. While the model currently in
development, based on the relationship between weather changes and incidence is expected to
predict outbreaks of vector-borne diseases, particularly malaria and diarrhoea, subsequently, it is
likely to monitor non-communicable diseases as well.
• There are certain diseases where weather patterns play a crucial role. Such as malaria, for which particular
temperatures and rainfall patterns can approximately predict whether an area is likely to have an outbreak
with fairly reasonable accuracy. Changes in rainfall and temperature patterns likely play a major role in
the increased incidence of these diseases across geographical locations.
• To verify the robustness of the advance warning system, a detailed analysis was carried out earlier this
year of malaria and diarrhoea cases in two districts of Maharashtra, Pune and Nagpur.
• An Intergovernmental Panel on Climate Change noted in its 2007 report that climate change may increase
the risk of diarrhoeal diseases, which is of major concern in developing countries, with increasing
incidents of floods as well as drought.
• Hence both the statements are correct.

Copyright © by Vision IAS


All rights are reserved. No part of this document may be reproduced, stored in a retrieval system or transmitted
in any form or by any means, electronic, mechanical, photocopying, recording or otherwise, without prior
permission of Vision IAS.Sparkle Copier

36 www.visionias.in ©Vision IAS

You might also like